Older Adult Final Review

Réussis tes devoirs et examens dès maintenant avec Quizwiz!

A clinic nurse assesses a client who has limited English -speaking ability. The child interprets for the client. Which action by the nurse is most appropriate? A) Obtain a professional interpreter. B) Talk directly to the interpreter. C) Teach the family member the appropriate medical terminology. D) Use the family member as a source for improving cultural competence

A A professional medical interpreter is preferred. The use of a child can influence the client's answers to questions. When using an interpreter, speak directly to the client.

The child of an 81 -year -old client asks the nurse about vitamins, antioxidants, and age -related macular degeneration. Which of the following theories of aging is most appropriate to this topic? A) Free radicals theory B) Immunosenescence theory C) Program theory D) Wear -and -tear theory

A Free radical theory focuses on interventions to modify or prevent the age -related accumulation of free radicals or to diminish the formation of free radicals. Antioxidants, beta -carotene, and vitamins C and E provide defense mechanisms against oxidative damage from free radicals. Immunosenescence (age -related decline of the immune system) theory focuses on the increased susceptibility of older people to diseases, such as cancer and infections. The immune system may even attack healthy cells, leading to autoimmune conditions, such as rheumatoid arthritis. Program theory postulates the life span of about 110 years in humans. Abnormal cells, such as cancer cells, are not subject to this predictable program and can proliferate an indefinite number of times. Wear -and -tear theory postulates that the longevity of the human body is affected by the care it receives, as well as by its genetic components

A nurse at a long -term care facility completes a minimum data set on each client. Which of the following categories are included in this assessment/plan of care? (Select all that apply.) A) Cognitive patterns B) Communication and hearing patterns C) Family support D) Mood and behavior patterns E) Psychosocial well -being

A,B,D,E Cognitive patterns, communication and hearing patterns, mood and behavior patterns, and psychosocial well -being are all categories within Minimum Data Set 3, and family support is not.

A nurse in an assisted living facility develops interventions that focus on improving cognitive abilities in the residents. Which of the following interventions should the nurse include in the plan? (Select all that apply.) A) Book club B) Calisthenics C) Christmas caroling D) Letter writing E) Reminiscence therapy F) Shopping trip

A,D The cognitive reserve model suggests that cognitive abilities can be improved through participation in creative and intellectually stimulating activities, such as art, storytelling, reading, writing, group discussions, and playing musical instruments. Calisthenics, singing traditional tunes, reminiscing, and shopping are not creative and intellectually challenging.

A nurse recognizes that depression has functional consequences. Which of the following are functional consequences of late -life depression? (Select all that apply.) A) Decreased functioning B) Dementia C) Higher incidence of a stroke D) Higher level of pain E) Increased risk for suicide

A,D,E Increased risk for suicide, decreased functioning, and higher level of pain are functional consequences of late -life depression. Strokes are a risk factor. Dementia is strongly related to depression but not a consequence.

An 89 -year -old adult is dismayed that his primary care provider referred him for a driving evaluation because he experiences vision problems and slower reaction time. Which of the following concepts is illustrated in this example? A) Risk factors B) Age -related changes C) Positive functional consequences D) Wellness outcomes

B Changes in vision and response time are considered to be age -related changes. These are not necessarily risk factors and they would not be considered positive functional consequences or wellness outcomes

A nurse is teaching nursing students about the importance of health promotion among older adults. Which of the following statements by a student indicates a need for further teaching? A) "Teaching older adults how to live with diabetes would be a useful health promotion initiative." B) "I can see why we would want to promote timely discharge back to the community following hospitalization." C) "I think that attending to spiritual growth could likely be a part of health promotion." D) "If we could promote healthy, simple diets, then some diseases could likely be prevented."

B Discharge planning is not a core component of health promotion. Health promotion denotes interventions or programs that focus on behavior changes that can improve health and well -being. Teaching older adults how to live with a particular disease, fostering spiritual growth, and promoting good nutrition are all components of health promotion

A nurse assesses the eating habits of a 75 -year -old client who takes iron supplements for iron deficiency anemia. Which of the following statements by the client indicates a need for further teaching? A) "I drink orange juice with my iron." B) "I prefer coffee to take my pills." C) "I take all my pills with a glass of warm water." D) "I take my iron in between my meals."

B Foods that change the pH of the gastrointestinal (GI) system interfere with the absorption of iron. It is best taken on an empty stomach, but if it causes GI upset, then it can be taken with orange juice, which helps absorption. Caffeine and some foods interfere with iron absorption. The temperature of the water should not impact the medication absorption

A nurse conducts a functional assessment of a client who has moved to the assisted living facility. Which of the following statements best describes this functional assessment? A) Information on the client's medical diagnoses and health problems. B) Client's ability to perform self-care tasks with a focus on rehabilitation. C) Assessment of the client's activities of daily living (ADLs). D) Prioritization of the client's ability to perform roles in relationships and in society

B Functional assessment is a way of determining an individual's ability to fulfill responsibilities and perform self-care. While it is distinct from a medical diagnosis approach, it does not discount or ignore information on an older adult's diagnoses and health problems. It includes data on ADLs and is not a counterpoint to ADL assessment. The focus is on the fulfillment of responsibilities and self-care more than on performing social and relationship roles.

A nurse assesses a 71 -year -old person who has smoked for 43 years. Which of the following is a negative functional consequence of smoking for this person? (Select all that apply.) A) Children are exposed to secondhand smoke B) Low oxygen -carrying capacity C) Abnormal breath sounds D) The ability to run a 5 -K race E) Pulmonary disease

B Functional consequences are the observable effects of actions, risk factors, and age -related changes that influence the quality of life or day -to -day activities of older adults. Low oxygen -carrying capacity, abnormal breath sounds, and pulmonary disease are negative functional consequences of smoking. The ability to run a 5 -K race is lost with smoking. Children may be exposed to secondhand smoke. These are not functional consequences of smoking for this person.

A nurse interviews a centenarian gathering data for a large study. In the interview, the centenarian defines aging as not growing older, but growing wiser. Which of the following is the best response to this definition by the nurse? A) "Aging might make you wiser, but it does lead to eventual death." B) "Healthy maturity is characterized by wisdom." C) "How did you get to live to this old?" D) "I will never make it to be 100 like you."

B Healthy maturity is characterized not only by physical decline but also by wisdom. The other responses are not therapeutic

A nurse plans culturally competent care for a variety of clients. Which of the following cultures is most strongly tied to the low health status? A) Hispanic in race B) Low socioeconomic status C) Member of LGBT society D) Resident of urban community

B Lower socioeconomic position is an overriding determinant of health status. Hispanic subgroups vary in rates of disability. While urban older adults and those in the LGBT community have special needs, their health is not significantly worse than others as a whole

A nurse discusses driving with an older adult who continues to drive, but is probably unsafe on the road. Which statement made by the nurse is most appropriate? A) I am calling your child to take your keys. B) I am concerned about your safety, as well as the safety of others. C) We just don't want you to crash when you drive across the state. D) You shouldn't drive anymore.

B Nurses can sensitively address issues about driving by expressing compassionate concern not only for the individual older adult but also for the safety of others.

A nurse reads up on some of the more common cultural groups in the local area. How should the nurse interpret the information that is available about cultural groups? A) Characteristics of cultural groups are normally consistent between every member of that group. B) Cultural generalizations can be useful and accurate, but they do not replace individualized assessment and care. C) It is simplistic and problematic to make generalized claims about members of a particular cultural group. D) It is unjust to categorize individual clients as being members of a specific cultural group.

B Nurses need to be knowledgeable about different cultural groups, but they need to use this information as a backdrop for exploring the ways in which individuals identify with the characteristics of the various cultural groups to which they belong. Generalized knowledge may be accurate and clinically useful, but it is not replacement for individualized knowledge. Nurses need to recognize that the culture of each individual person is based on his or her membership in many groups and is internalized in a unique and personal way

An 81 -year -old is admitted to the hospital for congestive heart failure. The client is widowed, and the medical staff and client are talking about the client moving to an assisted living facility. Which of the following interventions by the nurse best creates a wellness opportunity? A) Ask the client to explain how cares have been accomplished at home. B) Assist the client to discuss the feelings associated with a potential move to assisted living. C) Describe the options for long -term housing with the client. D) Encourage the client to think positively about this move

B Nurses promote wellness by asking older adults to talk about the meaning of life events that they have experienced. Asking about how cares have been accomplished does not assist the client. Nurses promote psychosocial wellness by encouraging older adults to express their feelings about decisions (not tell them how to feel) and help them identify effective ways of coping, even when they are not happy about the decision

A 75 -year -old woman who often used to go out to dinner with her friends has stopped going out because she has been experiencing urinary incontinence and is afraid of having an "accident" in public. When her child asks her why she doesn't go out with her friends anymore, she says, "I'm getting too old for such foolishness." Her child asks her to go to the doctor for an evaluation, but she refuses to do so. Which of the following is occurring with this older adult? A) She is experiencing learned helplessness and low self-efficacy. B) She sees incontinence as an inevitable consequence of aging. C) She views her incontinence as a negative functional consequence of aging. D) Her doctor is sympathetic; however, the woman and the doctor are unable to find a solution.

B Older adults may use a passive, emotion -focused coping mechanism and try to simply accept the situation. When older adults view functional decline as an inevitable consequence of aging, they are less likely to seek help for some treatable problems.

A nurse notes that a client who has heart failure could hear well during the last home visit, and is having difficulty hearing today. Which of the following laboratory findings is most likely associated with impaired hearing? A) Albumin of 4.1 B) Creatinine of 4.2 C) Potassium of 4.3 D) Sodium of 144

B Older adults with heart failure are at increased risk for hearing loss caused by medications (e.g., diuretics) and decreased renal and cardiac perfusion. Normal levels of albumin, potassium, and sodium are not associated with hearing loss.

A long -time resident of an assisted living facility has just been diagnosed with Alzheimer disease. A nurse who provides care at the facility has remarked to a colleague, "It's a real shame, but at least she'll never know what's happening to her." What fact should underlie the colleague's response? A) Older adults with Alzheimer disease and other dementias rarely have insight into their cognitive deficits. B) Many persons with dementia are acutely aware of the fact that they are experiencing a cognitive deficit. C) Certain types of dementia are occasionally marked by older adults' awareness of their disease. D) An awareness of dementia is an indication that the condition is either latent or resolving

B One of the myths associated with dementia is that people with dementia deny their symptoms or have no awareness of their deficits. In recent years, this perception of a high prevalence of so -called denial in people with dementia has diminished, and gerontologists are researching insight and self-awareness through all stages of dementia.

A 30 -year -old grandchild lives with and provides care for the 75 -year -old grandparent. The grandparent has congestive heart failure, hypothyroidism, and chronic pain from a compression fracture and osteoporosis. The grandchild supervises the older adult's medications. The home health nurse notes that the older adult has extra diuretic pills and that the pain medications for a month have been used and cannot be refilled for 2 more weeks. The older adult tells the nurse: "Those pain pills don't work, my back is always hurting." The nurse notes that the older adult's ankles are very swollen. Which of the following things should the nurse do first? A) Call adult protective services and ask for an immediate evaluation. B) Assess the grandchild's understanding of her grandmother's needs. C) Take the grandmother to the emergency department immediately. D) Tell the older adult that her grandchild is probably taking her pain medications.

B Physical neglect can arise from the caregiver's lack of knowledge. It is important to assess the caregiver's understanding of the dependent person's needs before drawing other conclusions.

A nursing home is in the planning stages of building a new wing that will be specifically designed for the needs of older adults who have dementia. What design characteristic should be included in this new facility? A) Monochromatic walls and floors that are a neutral color B) Pictures, signs, and color codes for identifying places C) Bright, glossy floors that can provide sensory stimulation D) Bright lighting during the day and total darkness at night

B Pictures, signs, and color codes can help to orient persons with dementia. Floors and walls do not need to be one color, and glossy floors and total darkness at night are safety hazards for this population.

A nurse knows teaching has been effective when the client states the following: A) "If my sensitivity to glare decreases and my contrast sensitivity increases, I will be evaluated for cataracts." B) "I wear sunglasses and a wide -brimmed hat when I am in the sun to protect my eyes and prevent the development of cataracts." C) "Having Alzheimer disease increases one's risk of developing macular degeneration." D) "If I take ototoxic medications, this will increase my risk for developing cataracts."

B Protecting one's eyes from sunlight is a health promotion intervention for preventing cataracts. Cataract symptoms include increased sensitivity to glare and decreased contrast sensitivity. Individuals with Alzheimer disease may have impaired contrast sensitivity and other visual impairments, but not an increased risk of developing macular degeneration. Ototoxic drugs will damage the auditory nerve; they do not affect the optic nerve or vision.

A nurse helps older adults increase healthy behaviors by restructuring. Which of the following communication techniques should the nurse use to help the older adult overcome barriers? A) "Could you walk with your friend for a half-hour after you both come back from the lunch program at the senior center?" B) "I know it's hard to get outside in the winter, so let's try to identify some ways of getting more exercise indoors during your usual activities." C) "Let's talk about the benefits of exercise. When you walk regularly you reduce constipation and muscle pain." D) "Your activity has been decreasing during the last 3 years and it is at the point that you are at an increased risk for heart disease."

B Restructuring assists the client to use positive thinking to focus on ways of overcoming barriers (e.g., getting outside in winter). Giving suggestions about when and where to walk might be helpful, but it is best if the client develops the plan of action. Talking about the benefits of exercise reinforces rewards. Discussing consequences of inactivity raises the consciousness of the older adult.

A nurse plans the care of older adults in a long -term care setting. Which of the following interventions incorporates the residents' connectedness to society? A) Ensuring that there are multiple television sets available to residents of the facility B) Arranging regular visits by schoolchildren to the facility C) Conducting reminiscence therapy D) Allowing residents to have input into the meal planning at the facility

B Social connectedness can be fostered by arranging meaningful contact between older adults and other members of society. TV, reminiscence, and input into routines may all have benefits, but none is likely to create a sense of connectedness with society

A home care nurse admits an older adult with macular degeneration. Which of the following assessment questions is most appropriate? A) "Do you have difficulty opening your medication bottles?" B) "How do you organize your medications?" C) "How many medications do you take each day?" D) "What medications do you take each day?"

B The client with macular degeneration will have limited sight; therefore, a question about assuring that this client takes the correct medications at the correct time is appropriate. There is no indication that this client would have difficulty opening bottles. Asking how many medications are taken each day is not helpful nor does it use therapeutic communication. The nurse will want to know what medications the client takes; however, this question is limiting. It does not include PRN, herbs, or even medications a client may take weekly. Nurses should ask additional questions about the client's ability to take his or her medications as prescribed based on specific observations.

An 85 -year -old woman who lives alone says to the nurse, "There is nothing I can do about my hearing. I am 85 years old, and I am not really interested in listening to television programs anymore." Which of the following would be the nurse's best response? A) "You are lucky you still live alone at 85, and I understand why you don't care about the programs on television." B) "Have you talked with your health care provider about a hearing evaluation? This would determine the problem and possible solutions to it." C) "I know a hearing aid dealer who offers free testing. Have you thought about trying a hearing aid?" D) "Did you know that there are closed -caption television sets that would allow you to enjoy some shows?"

B The first step would be to determine what the problem is. Free testing is not comprehensive in its evaluative scope. Telling the client she is lucky to be living alone at 85 years of age is nontherapeutic communication and suggesting that the woman use closed -caption television does not address the hearing issue.

A nurse is responsible for assessing an older adult in an acute care setting. Which of the following statements most accurately captures the complexity involved in assessing the older adult? A) Older adults manifest fewer symptoms of illness than do younger clients. B) Signs and symptoms of illness are often obscure and less predictable among older adults. C) Care must be taken to avoid assessing normal, age -related changes. D) Older adults experience fewer acute health problems but more chronic illnesses than do younger clients.

B The manifestations of illness in older adults can be less clear and less predictable than among younger clients. Older adults often show different, but not necessarily fewer, symptoms than do younger clients. Age -related changes must be recognized and acknowledged, not excluded from the assessment process. Older adults do not experience fewer acute health problems than do younger adults but rather different manifestations of health problems.

A nurse admits an 81 -year -old to the hospital for congestive heart failure. The client is widowed, and has recently moved to an assisted living facility. Which of the following contributed the most to this admission? A) Moving changed her daily habits. B) Her age -related changes and risk factors increased. C) The stress of widowhood and relocation stressed her body. D) The assisted living facility serves food high in saturated fats.

B There is a strong connection between chronic stress and health. Studies find that chronic stress increases the risk for onset of major illnesses and exacerbation of chronic illnesses

A nurse develops a plan of care for a family with nursing diagnosis of Caregiver Role Strain related to urinary incontinence. Which of the following interventions is the highest priority? A) Administer diphenhydramine at bedtime. B) Assist the client to the bathroom prior to bedtime. C) Limit the fluid intake of the client to 1000 mL each day. D) Monitor bowels for diarrhea and constipation

B Walking the client to the bathroom encourages voiding prior to bedtime. Urinary incontinence is a major factor that impacts placement in an institution. Fluid intake should be carefully timed and should be about 2,000 mL per day. Diphenhydramine relaxes bladder muscles, which can lead to urinary retention and incontinence.

A nurse is reviewing the side effects of antidepressants with a group of older adults. Which of the following statements by a member of the group indicates that the nurse's teaching has been effective? A) "I will start on the dose that I will take for life." B) "Fluoxetine should be given in the evening because it may help me sleep." C) "I need to maintain my fluid intake while on antidepressant medication." D) "The length of antidepressant treatment is usually 3 months for a first -time depression."

C An increase in fluid intake helps prevent the risk of postural hypotension. Dosages can be increased gradually until maximal therapeutic levels are reached, while observing for adverse effects. Fluoxetine should be given in the afternoon because of agitation. The length of treatment is usually 6 months for a first -time depression

A nurse counsels an older adult regarding nutritional requirements. Which of the following teaching points is priority when discussing age -related changes in nutritional requirements? A) "If possible, try to eliminate animal fats from your diet." B) "You should try to eat less meat and proteins than you did when you were younger." C) "Overall, you don't need to take in as many calories as you used to." D) "As an older adult, you don't need to eat as many starches and complex carbohydrates."

C Caloric requirements for older adults are significantly less than those for younger adults. It is unnecessary to wholly eliminate animal fats from the diet, and protein intake should remain same as for younger adults. Complex carbohydrates should constitute the majority of caloric intake

A healthy 65 -year -old says, "I don't think I will live much past 70." The studies however show that this client should live to 84 years of age. Which of the following statements, by the nurse, summarizes the compression of morbidity for this client? A) "Let's work on extending your life expectancy." B) "The goal is to live better, not longer." C) "We should work on postponing chronic illnesses." D) "You are lucky that you are healthy."

C Compression of morbidity emphasizes that preventive approaches must be directed toward preserving health by postponing the onset of chronic illnesses, but one's life expectancy cannot be extended. Consequently, disease, disability, and functional decline are "compressed" into a period averaging 3 to 5 years before death.

A nurse is leading a word -quiz game with a group of nursing home residents because the nurse knows this activity will assist the residents in maintaining: A) Fluid intelligence B) Adaptive thinking C) Crystallized intelligence D) Psychomotor memory

C Crystallized intelligence refers to vocabulary skills, information, and verbal comprehension. Fluid intelligence involves a person's inherent abilities, such as memory and recognition, and involves adaptive thinking. Memory involves retrieval and storage of information

A graduate nurse expresses an interest in focusing future study to healthy aging. Toward which of the following resources should the experienced nurse steer the graduate nurse? A) American Heart Association B) Healthy Brain Initiative C) Healthy People 2020 D) Springer Publishing Company

C Each of these is a possible source of data, but the Healthy People initiative is a well -known program that is a major source of recommendations for evidence -based health promotion interventions.

A nurse plans discharge for an older woman from China who is living with her son. Which of the following should be included in this plan to indicate that the nurse understands cultural influences? A) Include all family members in discharge planning. B) Ensure that the discharge plan does not include any technologic aspects. C) Review the discharge plans with the client and her son. D) Speak only with the client.

C In the Chinese culture, each family has a recognized male head who has great authority and assumes all major responsibilities. Avoidance of technology is not part of this culture

An older adult who appears to be between 85 and 95 has been brought to the emergency department by emergency medical services after being found wandering in the street. The older adult is filthy, confused, and exhibits numerous bruises to the face and neck as well as signs of malnutrition and dehydration. What problem should the nurses prioritize for assessment and intervention? A) Hygiene B) Malnutrition C) Dehydration D) Potential elder abuse

C Nursing guidelines emphasize that interventions related to hydration status are higher priority than most other problems, including hygiene, and malnutrition; elder abuse is not an immediate threat when the older adult is in a health care setting

A nurse evaluates the plan of care for a client who experienced an ischemic stroke. Which of the following assessment findings should signal the nurse to the possibility that the client has developed dysphagia? A) The client complains of being excessively hungry. B) The client drinks large amounts of water with meals. C) The client pockets food in the affected cheek during meals. D) The client prefers to sit in a high Fowler's position after eating

C Pocketed food suggests dysphagia. Sitting upright after meals prevents, rather than indicates, dysphagia and neither hunger nor high fluid intake is indicative of dysphagia.

During an admission interview, a client gives the following response to a question about living arrangements. "I can't stay in my own home. Now that I've fallen and broken my hip, I'm not sure what the doctor will say. My children don't want me." Which response by the nurse is most appropriate? A) "You worry that the doctor will tell you need surgery?" B) "You fell and broke your hip?" C) "Your children don't want..." D) "Where you want to live?"

C Reflection about the children gives feedback about what the nurse heard and leads into further questions about underlying feelings. The client has already expressed that they can't live at home, asking where they want to live when they don't have the choice is not therapeutic. We know that this client has a broken hip, and the conversation is not about surgery.

A nurse teaches an older adult client about the use of the telehealth equipment to monitor congestive heart failure. Which of the following statements by the client shows understanding? A) "I will call the primary health care office everyday with my weight, and blood pressure." B) "I won't touch this fancy equipment unless you are here." C) "I need to step on this scale and use this automatic cuff each day." D) "I will watch the prescribed television show every afternoon."

C Telehealth is used to collect and transmit assessment information. The client does not have to call anyone, they are to use the equipment to collect weight and blood pressure to monitor congestive heart failure, this equipment will transmit the data. Television is not included in telehealth

A nurse is teaching a group of hearing -impaired nursing home residents about hearing aids. Which of the following points should the nurse emphasize? A) It is not necessary to use the hearing aid for one -on -one conversations. B) The hearing aid should be used only in the dining room or social area. C) While inserting the hearing aid, make sure the volume is turned off. D) If whistling is heard, the volume of the hearing aid may need to be increased.

C The hearing aid should be inserted with the volume off with the canal portion pointing into the ear. A hearing aid should be used for one -on -one conversation and should not be used in a dining room where there is background noise. If whistling is heard, the volume should be decreased

A 70 -year -old tells a nurse, "I am worried that I'm losing my mind, I have difficulty remembering names as well as I used to, and I missed two health care appointments in the past month because I forgot about them." The nurse initiates a memory training program, although the nurse has been unable to identify any risk factors that might affect the older adult's cognitive abilities. Which of the following questions is the best approach to evaluating the effectiveness of the memory training program? A) "Have you seen an improvement in your memory?" B) "Are you less worried about your memory now?" C) "How have the memory training techniques helped you?" D) "Are you using the memory training techniques now?

C The question, "How have the memory training techniques helped you?," allows the older adult to tell the nurse how memory training has helped and is more open -ended than the other options. It also communicates positive expectations. The question helps identify the techniques that are most effective for the individual

A nurse reviews the medication list of an older adult. Which of the following age -related changes leads to a slower drug clearance? A) Increase in sensitivity to bioactive substances B) Elimination half-time is likely to be faster. C) Increase in glomerular filtration rate D) Decrease in hepatic blood flow

D The older adult is not more sensitive to bioactive substances. Elimination is likely to be slower. There is an age -related decline in hepatic blood flow that impacts pharmacokinetics, as well as the glomerular filtration rate.

Until recently, a 77 -year -old client lived alone in her own home. The client fell and fractured an ankle and was placed in a long -term care facility for physical therapy. After the physical therapy was finished, the client tells the nurse, "I want to stay at the facility; I am happy living there and I like the social interaction." Which of the following theories of aging best describes the status of this client? A) Activity theory B) Feminist theory C) Life -course theory D) Theory of thriving

D The theory of thriving posits that the older adult thrives when there is concordance between the person and the human and nonhuman environment. Activity theory postulates that older people remain socially and psychologically fit if they remain actively engaged in life such as engaging in full -time work and low -level volunteering. Feminist gerontology theories examine aging from the experiences of older women. Feminist theories address gender inequalities with regard to caregiving roles, diseases, and economic status. Life -course theories address old age within the context of the life cycle

A nurse councils a care partner of a client with dementia. The care partner states "He fights me when I try and bath him; he hasn't had a shower in 2 months!" Which response by the nurse is most appropriate? A) "I hear your frustration." B) "He wants to feel he has a choice. How do you get him to shower?" C) "I would just put him in there, he needs to be clean." D) "Whatever worked before should work now." E) "What other ways have you tried to assure he is clean?"

E There are multiple ways to stay clean, if showering is a trigger, then avoid it. What has worked in the past does not mean it will work again. Forcing a shower is unsafe. They don't get him to shower, it's been months.

A graduate nurse expresses an interest in focusing future study to healthy aging. Toward which of the following fields should the experienced nurse steer the graduate nurse? A) Palliative care B) Gerontological nursing C) Nursing home administration D) Social work

B Gerontological nurses are the health care professionals consistently responsible for the 24 -hour care of older adults in all clinical settings

A gerontological nurse is aware that quality of life is an important consideration when assessing the functioning of older adults. What measure should the nurse use when appraising older adults' quality of life? A) Active life expectancy B) Gerotranscendence C) Life expectancy D) Rectangularization of the curve

A Active life expectancy, which is measured on a continuum ranging from inability to perform activities of daily living to full independent functioning, is an indicator of quality of life during later adulthood. The theory of gerotranscendence addresses the change in perspective that often accompanies aging, and life expectancy is a measure of chronologic age. Rectangularization of the curve is attributed to changes in survival caused by various significant factors occurring at different points in time.

A 76 -year -old adult expresses frustration to the nurse regarding hearing loss despite a lifetime of being conscientious about avoiding known causes of hearing damage. Which of the following age -related changes may result in hearing loss? A) Degeneration of the inner ear structures B) Decreased viscosity and quantity of cerumen C) Plaque formation and occlusion of the Eustachian tubes D) Hypertrophy of the external ear structures

A Age -related changes of the inner ear include loss of hair cells, reduction of blood supply, diminution of endolymph production, decreased basilar membrane flexibility, degeneration of spiral ganglion cells, and loss of neurons in the cochlear nuclei. These inner ear changes result in the degenerative hearing impairment termed presbycusis. Cerumen often becomes more viscous with age, and occlusion of the Eustachian tubes is not a normal, age -related change. Changes to the external ear structures are not implicated in age -related hearing loss

An 85 -year -old client takes meals on wheels around the community. The client states, "All those old people really need me, you know how older people are. They can't get out, and are a burden to their family, and I just want to help." Most of the people on the client's delivery route are in their 60s. Which of the following characterizes this scenario? A) Ageism B) Aging anxiety C) Aging attribution D) Antiaging

A Ageism is a way of pigeonholing people and not allowing them to be individuals with unique ways of living their lives. Older adults between the ages 81 and 98 held more ageist stereotypes and reported more avoidance of older adults than younger older adults. The antiaging movement views aging as a process that can be stopped and the life span as something that can be extended for up to 200 years. Aging anxiety is fears about detrimental effects associated with older adulthood. Age attribution is the tendency to attribute problems to the aging process rather than to pathologic and potentially treatable conditions.

An older adult with heart failure and mild dementia states the intent to refuse low sodium diet and diuretics, stating: "It's important to me to live free, without restrictions on what I eat." The family is supportive. Which action, by the nurse, should be done first? A) Assure that the client understands the consequences of this decision. B) Discuss this decision with the older adult's family to plan for the future. C) Document the client's wishes in the plan of care. D) Notify the primary health care provider of the client's wishes.

A All of these actions should be done; however, the primary concern is that the nurse assess the client's understanding of the consequences. During mild -to -moderate stages of dementia, assessment of decision -making ability is based on the person's ability to describe the importance or implications of the choice on his or her future health. Medical decision making is a complex process in which information is shared between clients and clinicians and among family and others who are affected by the outcomes

An older adult reflects, "Why should I go to the gym, I'm going to get fat anyway." Which response by the nurse is most appropriate? A) "Age -related changes are inevitable; however, most problems affecting older adults are related to risk factors, so it's important to do what you can to maintain a high level of functioning." B) "Older adults experience positive or negative functional consequences because of age -related changes." C) "Risk factors do impact consequences, but you can override them." D) "Many problems affecting older adults are based on genetics."

A Although age -related changes are inevitable, most problems affecting older adults are related to risk factors. Older adults experience positive or negative functional consequences because of a combination of age -related changes and additional risk factors. Interventions can be directed toward alleviating or modifying the negative functional consequences of risk factors.

A nurse administers medications to a group of older adults in a residential facility. Which of the following clients is most likely to experience adverse effects? A) A 77 -year -old man with a creatinine of 3.6 B) A 78 -year -old man with a body mass index of 35 C) An 84 -year -old woman with iron deficiency anemia D) An 82 -year -old woman with constipation

A Although age -related changes can influence skills related to taking medications, risk factors that commonly occur in older adults exert a stronger influence. A creatinine of 3.6 reflects renal failure, which will lead to increase in serum levels of medications. Iron deficiency anemia, obesity, and constipation do not impact the risk of adverse and altered effects

A nurse assesses the urinary elimination of older adults. Which of the following actions by the nurse is most appropriate? A) Work to identify terms that the older adult comprehends. B) Wait until the person initiates a discussion of this embarrassing topic. C) Give the interview questions to the client in writing. D) Ask the older adult to keep a urination diary

A Although nurses usually learn to discuss urinary elimination with relative ease, older adults may feel uncomfortable with the topic, especially if there are gender or age differences between the older person and the nurse. In addition, older adults may be reluctant to discuss urinary problems because they tend to accept urinary leakage as an inevitable consequence of aging and gradually increase their tolerance threshold. Because of varying social contexts, successful interviewing about urinary elimination and incontinence depends on identifying the terms that are least embarrassing and most understandable to the older adult.

A nurse assesses a 70 -year -old man who has high blood pressure and chronic obstructive pulmonary disease (COPD). He has been prescribed nicardipine and ipratropium inhaler. This medication combines a calcium -channel blocker and an anticholinergic. For which of the following urinary effects should the nurse teach the client to monitor? A) Nocturia B) Urinary tract infection (UTI) C) Urge incontinence D) Hematuria

A Calcium -channel blockers decrease bladder contractility and can cause urinary retention, frequency, nocturia, and incontinence. Anticholinergic agents decrease bladder contractility, and relaxed bladder muscle can cause urinary retention, frequency, and incontinence.

A nurse uses the Functional Consequences Theory to assess older adults. Which of the following situations best demonstrates the effect of physical environment on the older adult? A) A resident of a care facility experiences a fall because there are not grab bars outside his bathtub. B) A hospital client develops Clostridium difficile -related diarrhea because a care provider did not perform adequate handwashing. C) An older adult cannot afford a wheeled walker and suffers a fall while trying to ambulate using a cane. D) An assisted living resident requires care for emphysema that resulted from a 70 pack -year history of cigarette smoking.

A An adverse health effect that results from the inadequacy of one's surroundings (such as the lack of safety devices) is an example of the domain of environment. Although the use of incorrect equipment, health problems caused by lifestyle factors, and infections that result from caregiver negligence create risk factors for older adults, these problems are not situated within the domain of environment.

A nurse hears a colleague make the statement, "Most older adults have nothing to worry about financially." Which of the following responses is most appropriate? A) "You have to remember that there's a huge economic disparity among older adults." B) "Actually, the number of older people living below the poverty line has been increasing, not decreasing." C) "This isn't really true now, but it is true that the gaps that disadvantaged groups live with are expected to shrink." D) "This is true for some groups, but not for minorities who are less likely to be living with their relatives."

A An important consideration with regard to economic conditions of older adults is the tremendous range in financial status, which varies significantly according to race, gender, and living arrangements. The number of adults living below the poverty line, however, has decreased in recent years. The disparities that separate disadvantaged groups from more economically secure older adults are predicted to persist. Minorities are more likely, not less likely, to be living with their relatives

A nurse admits a 90 -year -old client to the hospital with a diagnosis of failure to thrive. Which of the following laboratory data should the nurse expect? A) Low albumin and red blood cells B) Elevated white blood cells (WBCs) and low potassium C) Low platelets and low prothrombin time (PT) D) Elevated calcium and magnesium

A Anemia and low serum albumin levels are consistent with malnutrition. Elevated WBCs, calcium, and magnesium and low platelets and PT are not characteristic of malnutrition

An older adult has impaired psychosocial functioning. Which of the following consequences should the nurse monitor? A) Anxiety B) Elevated blood glucose level C) Increased independence D) Resilience

A Anxiety is a common result of impaired psychosocial function in older adults. It is less likely to result in hyperglycemia and it is not associated with increased independence or resilience, another positive consequence of healthy psychosocial functioning

A nurse monitors older adults in a long -term care facility. Which of the following symptoms would require follow -up by the nurse to assess for depression in the older adult? A) Anorexia B) Weakness C) Labile affect D) Impaired perceptions

A Appetite disturbances, particularly anorexia, are among the most common physical complaints of depressed older adults. Individuals with dementia have the following symptoms: vague fatigue, labile affect, and physical complaints that are easily forgotten.

A nurse addresses the social supports available for an older adult client. Which of the following should the nurse include in the plan? A) Ask the client direct questions about the barriers to the use of social supports. B) Decide which of the programs is the highest quality. C) Determine if family or friends could do the work. D) Provide the client information about services that are available

A Assessing barriers to support services is particularly challenging because direct questions about these issues often are inappropriate and usually are very threatening. Social supports are often erroneously seen as expensive, impersonal, and hard to arrange. Funding is available for such programs, though less than 20% of eligible adults use them. The quality of care is not necessarily lower than that provided by friends or family and an older adult is not disqualified by virtue of having friends or family available

A neighbor notices an 81 -year -old getting water from someone's outside faucet. The neighbor notices that this person's ankles are very swollen and there is an open wound on her left leg. The older adult says, "I stopped taking my pills because the water department turned off my water and I can't use the bathroom. My daughter did not pay the water bill, and she never has time to take me to the doctor so my legs can be checked." The neighbor calls adult protective services. Which of the following interventions is the priority when the nurse visits for an evaluation and does not find any immediate danger? A) The competency of the older adult in making decisions needs to be determined. B) The daughter needs to be picked up by the police on a neglect charge. C) The older adult needs to be involuntarily committed to a long -term care facility. D) An involuntary legal intervention needs to be initiated immediately.

A Because the older adult is not in immediate danger, the first step is to determine her competency and the ability to make decisions for herself.

A resident of a nursing home has experienced a progressive loss of vision over the past several months as a consequence of diabetes. How should the nurse accommodate the resident's loss of visual acuity? A) Provide the resident with brightly colored grooming utensils. B) Replace the resident's tube television with a flat -screen TV. C) Remove books from the resident's room to avoid reminding her of her vision loss. D) Have the walls in the resident's room painted a neutral color that matches the color of the flooring

A Brightly colored utensils and grooming supplies can make it easier for an older adult to identify and use them. Replacing the TV is not an effective intervention, and it is not appropriate to remove books simply because they may remind the resident of her vision loss. Contrasting, not monochromatic, color schemes facilitate vision.

Centers for Medicare and Medicaid Services (CMS), the Institute of Medicine, and the Joint Commission have developed standards to address areas of concern for older hospitalized adults. Which of the following situations is of particular concern for an older adult with a hospitalization requiring complex care? A) Transitions in care B) Hospital -acquired respiratory infections C) Need for geriatric care manager D) Placement in an acute care for elders unit

A CMS, the Institute of Medicine, and the Joint Commission have placed a high priority on the issue of older adults with complex medical problems who transfer between care settings, because they are particularly vulnerable to experiencing problems. Geriatric care managers and acute care for elders unit assist with this issue. Hospital -acquired urinary tract infections and wounds not respiratory infections are prevalent.

Active care management is often necessary in order to maintain wellness among older adults. Which of these older adults is most likely to require care management? A) A 90 -year -old man who lives alone and has no living family members B) A 77 -year -old woman who enjoyed good health until she suffered a severe stroke 3 days earlier C) An 81 -year -old resident of a nursing home whose Alzheimer disease is progressing rapidly D) A 90 -year -old man who has recently been transferred from an assisted living facility to an acute care setting

A Community -dwelling older adults who may lack family involvement in their care often require independent community -based professional geriatric care management. Individuals who are experiencing acute medical conditions and who are in institutional or acute care settings are not frequent recipients of care management

A nurse teaches a nursing assistant about the impact of culture on older adults' well -being. Which of the following statements by the nursing assistant indicates a need for further teaching? A) "A cultural background has little influence on individuals' standards for 'normal' or 'abnormal' behavior." B) "Western cultures often have a very rigid distinction between health and illness." C) "Culture may influence mental health and illness in individuals." D) "Culture may determine an individual's expression of symptoms or clinical manifestations."

A Cultural background significantly influences how a person defines all aspects of psychosocial function. It is essential to recognize that every society has standards of behavior. Many societies do not have the rigid distinction between health and illness that Western society does

A resident of a nursing home has accused several members of the care staff of stealing jewelry from the overbed table despite the fact that the facility's policy requires residents to keep such valuables in a lock box. The nurse has responded empathically to the accusations and has explained why this is impossible, to no avail. Which of the following conditions is the client experiencing? A) Delusions B) Hallucinations C) Unresolved anger D) Illusions

A Delusions are fixed false beliefs that have little or no basis in reality and cannot be corrected by appealing to reason. Hallucinations are sensory experiences that have no basis in an external stimulus. Delusions are not known to be a manifestation of unresolved anger. Illusions are misperceptions of an external stimulus

A gerontological nurse is aware that out -of-pocket expenses for care can be onerous for many older adults. Which action can the nurse take to potentially minimize these expenses for clients? A) Become familiar with the various funding sources and their eligibility requirements. B) Teach older adults to be astute with their spending and saving patterns. C) Encourage older adults to make care providers aware of each chronic condition they live with. D) Provide care that is primarily focused on acute, rather than chronic, health problems.

A Despite the complexity and limitations of programs, nurses need to know enough about the most common sources of payment for health services so they can understand and address some of the barriers to and challenges of implementing nursing care plans and discharge plans. Ultimately, this may have the effect of reducing some older adults' out -of-pocket expenses. Teaching about financial management is beyond the scope of the nurse and focusing on acute, rather than chronic, health problems is inappropriate

A nurse assesses risk factors for vision loss in a 71 -year -old client. Which question should the nurse include in this assessment? A) "Do you have high blood pressure or diabetes?" B) "Did your parents wear glasses or have cataracts?" C) "How much red meat do you usually eat?" D) "Do you have high cholesterol?"

A Diabetes and hypertension are significant risk factors for vision loss. Family history, diet, and high cholesterol are not closely associated with vision loss in older adults.

A nurse at a rehabilitation unit assesses an 86 -year -old woman with a BMI of 30 and a history of heart failure, whose oral intake is declining. Which of the following risk factors is related to this older adult's decline in appetite? A) Diuretics B) Exercise C) Female gender D) Obesity

A Diuretics decrease saliva, olfactory function, and gustatory functioning. Women have better olfactory and gustatory function than men; exercise increases appetite. Obesity is unrelated

Which of the following statements by residents of a nursing home should prompt a nurse to assess for depression? A) "Lately I wake up for the day at 4:00 or 5:00 in the morning and can't fall asleep again." B) "I've got these cravings for sugary and salty snacks more than I used to." C) "I've never been too prone to headaches, but these days I always seem to have one." D) "I don't know why this sore on my ankle just won't heal this time."

A Early morning waking is a sleep disturbance that is characteristic of depression. Headaches and impaired healing may also be linked with depression, but sleep disturbances are more highly associated with the problem. Food cravings are not typical of depression in older adults.

A diabetes nurse is providing care for a 73 -year -old client who is a regular client of the hospital's out -client diabetes clinic. What assessment question most clearly addresses this client's potential for optimal function? A) "What are some goals that you have for maximizing your level of wellness?" B) "How can we help you to take ownership of your own health?" C) "Is there anything that you're doing that might be exacerbating your diabetes?" D) "How long do you think that you'll be able to live independently?"

A Eliciting an older adult's goals for wellness and high functioning can help promote these outcomes. Questions about a client's living situation, disease management, and personal responsibility may or may not promote wellness.

A nurse is conducting a class at a senior citizens' center on factors that protect against dementia. Which of the following statements by an older adult in the class indicates a need for further teaching by the nurse? A) "No healthy lifestyle is going to ward off dementia." B) "Eating food high in omega -3 fatty acids will help preserve my thinking processes." C) "Engaging in social activities will help prevent dementia." D) "Engaging in an organized exercise program will help prevent symptoms of dementia."

A Engaging in regular exercise has a positive effect on preventing cognitive decline. Omega -3 fatty acids are nutritional interventions that help preserve cognition. There is evidence that engaging in stimulating and meaningful activities also has a positive effect on preventing cognitive decline. Adopting a healthy lifestyle may help ward off dementia just as it does other diseases

A rural community nurse sets up a monthly disease screening service. Local news coverage chooses not to cover this event. Which of the following justifications best supports the screenings? A) Health promotion is essential for older adults because they have complex health care needs. B) Older adults are less responsive to health promotion interventions. C) Older adults as a group receive fewer prevention and screening services than other populations. D) Preventive services are less effective after the onset of chronic illness.

A Even though health promotion interventions are cost -effective ways of preventing disease and disability and improving functioning and quality of life for older adults, older adults as a group receive fewer prevention and screening services than other populations. This is due to misperceptions, such as (1) older adults are less responsive to health promotion interventions, and (2) preventive services are less effective after the onset of chronic illness. In reality, health promotion is essential for older adults precisely because they have more chronic conditions, have complex health care needs, and use considerably more health care services than younger adults

A nurse assesses an 85 -year -old Hispanic woman. The client states that her husband was punished by God. To which of the following illnesses is the woman most likely referring? A) Alcohol abuse B) Fainting C) Posttraumatic stress disorder (PTSD) D) Voodoo

A For some Hispanics, mental illness may be viewed as a punishment by a supreme being for past transgressions; Hispanic older adults define mental health problems as alcohol and other drug abuse. PTSD is relatively common in immigrants. Hallucinations are not especially related to Hispanic culture. Those of Caribbean descent may attribute the cause of mental illness to voodoo

A nurse leads a "Healthy Aging" class at a community health center. Which question should the nurse use to generate discussion among participants in this setting? A) "How did you adjust to your move from your house to the assisted living facility Irma?" B) "Are you satisfied with the care that you're getting from your family doctor, Elizabeth?" C) "Donald, could you tell us why your grandson is living with you?" D) "Have you had any tests done on your heart since we last met, Marie?"

A Healthy aging classes are based on the belief that older adults who are beginning to recognize age -related physical and psychosocial changes or who are already dealing with such changes can benefit from sharing their experiences with their peers. Discussion about these adjustments should be the priority in a healthy aging class

A nurse is conducting a comprehensive psychosocial assessment of an older adult who has recently moved to the long -term care facility. How should the nurse best assess the client's motor function? A) Observe the client walking into or out of the room. B) Assess the client's deep tendon reflexes using a hammer. C) Perform passive range of motion exercises on the client's arms and legs. D) Position the client supine and ask the client to perform a leg lift with each leg separately.

A In the context of a psychosocial assessment, motor function includes such assessment parameters as posture, movement, and body language. It is not an assessment of reflexes, strength, or range of motion

An older adult expresses frustration about limitations of aging. Which of the following statements by the nurse promotes wellness? A) "Do you have some words of wisdom to share about that valuable experience?" B) "How does living in these conditions compare to your youth?" C) "Have you met any of your neighbors, they seem like nice people?" D) "What you are saying is that you are frustrated with how they are not listening to you?"

A Instead of asking about current versus historical, the nurse should acknowledge the wisdom of older adults by asking questions such as "Do you have some words of wisdom to share? Asking about neighbors is deflection and not helpful?" Reflection can be appropriate, but at this time positive acknowledgment should be used.

Which of the following statements best explains the relevance of psychological theories for gerontological nursing? A) Human needs theory allows the nurse to determine priorities of nursing care for older adults. B) Life span development theories support the belief that it may be difficult to initiate behavioral changes in older adults. C) Psychological theories explain why nurses should focus their discussion more on the present than on the past when talking with older adults. D) Psychological theories explain why reminiscence groups may not be beneficial for older adults

A Maslow's human needs theory is useful in conceptualizing interventions in the older adult's home and in a health care facility. The attainment of lower -level human needs takes priority over higher -level human needs, such as self-actualization. Life span development theories help nurses identify those areas of personality that are likely to change and those that are more likely to remain stable. Psychological theories imply that older adults should devote some time and energy to life review and self-understanding

A nurse observes an aide asking a client what he wants for breakfast, lunch, and dinner while assisting him to toilet. Which action by the nurse is most appropriate? A) Direct the aide to present only one idea at a time. B) Encourage this small talk. C) No action is required. D) Tell the aide to avoid conversations while the client is toileting

A Multiple events/ideas are occurring at once; this increases the risk of overloading the client. Teach the aide to present one idea at a time (e.g., do you like oatmeal?). Directed small talk is appropriate, when the client is dressed and not in a compromised situation.

A nurse notes that an older adult is unable to process complex thoughts and has difficulty forming sentences. Which of the following actions by the nurse is priority? A) Review medication administration record. B) Place the client on high fall risk precaution. C) Assess muscle strength and deep tendon reflexes. D) Orient the client to environment.

A Nurses need to be alert to the possibility that even a simple over -the -counter product is a common cause of mental changes in older adults. An acute confusional state can be precipitated by any medication or by medication interactions. Assessment of deep tendon reflexes will be important to assess fluid and electrolyte imbalances; review the medications first. Orienting the client and high fall risk may be needed (or not).

A 76 -year -old Hispanic woman has been admitted to the hospital. There are no advance directives in the chart. Which of the following questions will best prepare the nurse to begin a dialogue with this client about advance directives? A) "With whom do you talk to about your health care decisions?" B) "I see that you have no advanced directives on your chart, could you tell me about that?" C) "Is there someone we should call to join us while we discuss your care?" D) "Tell me about your living arrangements; do you live alone or with others?"

A Nurses need to identify patterns of culturally influenced decision making in order to ask questions and obtain relevant information from clients of different cultures. Why there are no advance directives on the chart is not the place to start to open the conversation. Whether or not she lives with someone, inviting family to join reflects an emphasis on family caregiving consistent with the Hispanic culture

A group of nurses is involved in the planning and implementation of a health promotion campaign aimed at older adults. Which of the following questions is the best guide to such a campaign? A) "How can we help older adults maintain wellness as they age?" B) "What can we do to increase life expectancy in our region?" C) "How can we help older adults avoid age -related changes?" D) "What is stopping older adults from living longer lives?"

A Nursing practice prioritizes the maintenance of wellness into later life rather than simply increasing chronological life expectancy. It is unrealistic to expect older adults to avoid age -related physiologic changes.

A nurse teaches an older adult about changes to nutritional requirements. Which of the following meal choices would give evidence that the older adult understands the teaching? A) Baked chicken, carrots, and angel food cake B) Green salad, mashed potatoes, and an oatmeal cookie C) Vegetable beef soup, crackers, and Jell-O D) Baked pork chop, green beans, and sherbet

A Older adults need increased intake of foods with a high nutritional value and a concomitant decrease in the intake of foods containing little or no nutrients

A nurse teaches an older adult about risks related to ototoxic medications. Which of the following medications should the adult minimize or avoid? A) Nonsteroidal anti -inflammatory agents B) Osmotic stool softeners C) Over -the -counter sleep aids D) Penicillin -type antibiotics

A Ototoxic medications include (but are not limited to) aminoglycosides, macrolides, quinolones, and some antifungals (not penicillins), aspirin and other salicylates, as well as nonsteroidal anti -inflammatory drugs. Neither sleep aids nor stool softeners have been implicated in ototoxicity

A nurse is teaching an older adult about possible involvement in Programs of All -inclusive Care for the Elderly (PACE). Which of the following statements by the older adult shows understanding? A) PACE programs provide several social and medical services on a managed care basis. B) PACE programs provide a cost -effective alternative to hospital -based acute care. C) PACE programs are more expensive than fee -for -service models but offer better health outcomes. D) There is pressure for Medicare and Medicaid to begin funding PACE programs.

A PACE programs provide a range of services using a capitated managed care model. They are focused on meeting the needs of adults with chronic conditions and are not an alternative to in -hospital treatment of acute illness. They are less expensive than fee - for -service models and presently are receiving funding under both Medicare and Medicaid. The 2010 Affordable Care Act provides incentives for further expansion of PACE programs.

A 93 -year -old asks the nurse, "I sure would like to live to get that 100 year birthday card from the president." Which of the following responses by the nurse is best? A) "Keeping fit and dealing with stress in a positive way helps your chances of living to be 100." B) "Surviving to 100 is strongly impacted by eating meat, fruits and grains." C) "Those people in your socioeconomic situation have higher chance of living to 100." D) "You have had a cancer and a stroke, so that decreases your chance of surviving to 100.

A People who survive to 100 years and older are a heterogeneous group with a wide range of health and socioeconomic characteristics. Most centenarians have escaped the common pathologies, such as stroke, cancer, and myocardial infarction (Andersen, Sebastiani, Dworkis, et al., 2012; Vacante, D'Agata, Motta, et al., 2012). Variables commonly identified as predictors of healthy longevity include nutritional patterns with a high intake of plant -based foods and being resilient in the face of stress (Davinelli, Willcox, & Scapagnini, 2012; Hutnik, Smith, & Koch, 2012)

A nurse develops a plan of care for an older adult recently diagnosed with Lewy body dementia. Which functional consequence would be most important to monitor in this older adult? A) Development of visual hallucinations B) A visual acuity score of 20/30 C) Improved visual acuity after medications for dementia D) Growth of cataracts

A Persons with Lewy body dementia are at risk for visual hallucinations. Low vision is 20/70 visual acuity to 20/200 visual acuity. Visual acuity will not improve with dementia medications, as anticholinergics also impair vision. Cataracts are unrelated to Lewy body dementia, although they are common in all older adults

Which of the following is a priority nursing intervention for the management of delirium? A) Giving the client low -dose oxygenation and maintaining his or her fluid and electrolyte balance B) Reducing noise and placing familiar objects in the client's environment C) Giving the client a clock, a watch, and calendars to provide the client with temporal orientation D) Providing psychological support through cognitive and social stimulation

A Priority questions address physiologic integrity. The client needs to be stabilized before the other interventions can be implemented.

A nurse manager develops policies to promote a sense of control for older adults in the assisted living facility. Which of the following policies should be included? A) Hold resident council meetings twice monthly and invite all residents to attend. B) Post a meal menu every Sunday and tell the residents that they must notify the kitchen in advance if they want a menu change. C) Design all the emergency pull cords so they blend in with the wallpaper and are inconspicuous. D) Teach the nurses' aides to use the passkey to do spot checks on every resident at least twice during the night to ensure that the residents are safe

A Resident meetings allow older adults to address personal preferences and to make choices. Posting the meals and later allowing choices is giving the older residents a limited chance to make a choice. Safety should be an ongoing concern

A wound care nurse is assessing a 76 -year -old client. The client has intimated to the nurse that her son sometimes "flies off the handle and gets rough with me." Which response made by the nurse is the best response? A) "When you say 'gets rough,' what does that look like?" B) "What do you think usually provokes this to happens?" C) "I'm going to have to phone adult protective services right now." D) "Why do you think that there is that response with anger or frustration?"

A Safety is the first priority in cases of elder abuse and prompt action is often necessary. However, gathering additional information, detail, and context is appropriate when a threat is not immediate. Speculating about a perpetrator's motives is unnecessary and inappropriate

A nursing case manager monitors admissions into an acute care unit. Which of the following clients would be the most appropriate candidate for in -home skilled nursing care? A) A client requiring twice -daily dressing changes for a coccyx wound B) A client who has been admitted to the emergency department with a recent stroke C) A client with reoccurring urinary retention of unknown etiology D) A client who is scheduled for hip replacement surgery tomorrow

A Skilled home care is most appropriate for older adults who are recovering from an illness or injury and have potential for returning to their previous level of functioning. Following a stroke, a client requires hospitalization. A client with a poorly understood or undiagnosed health problem would not be an ideal candidate for home care, nor would a preoperative client

A nurse plans care for a client who states that food is no longer appealing. The nurse notes a dry mouth and teeth in poor condition. Which interventions should the nurse include in the plan of care? (Select all that apply.) A) Eight -ounce bottle of water between each meal B) Hard toothbrush C) Ice cold water at bedside D) Meals in the common room E) Oral care before each meal

A Social isolation can lead to lack of appetite. Saliva -producing activities before each meal and 60 to 80 ounces of water a day are recommended to treat dry mouth. Iced drinks are less palatable to the older adult with poor oral condition. A soft electric toothbrush is recommended.

A nurse manager of an extended care facility works to promote psychosocial health. Which of the following interventions should the nurse manager include? A) Adapt the environment to compensate for residents' sensory impairments. B) Dress residents exclusively for ease in going to and from the restroom. C) Plan dining room arrangements according to room and hall assignments. D) Position the residents who are in wheelchairs solely for ease in getting out of the dining area

A Table and room arrangements should be made in a way that promotes social relationships. Older adults should be allowed to choose between at least two alternatives when dressing. Residents in wheelchairs should be positioned to promote social interaction

A nurse plans activities each month at an assisted living facility. Which of the following activities is most cognitively stimulating? A) Book discussions B) Movie night C) Exercise D) Reminiscence therapy

A The cognitive reserve model suggests that cognitive abilities can be improved through participation in creative and intellectually stimulating activities. Reminiscence may provide some social interaction and movies serve as a distraction. Exercise does increase the blood flow; however, the brain and neural circuits develop in response to environmental stimuli (neuroplasticity).

An 80 -year -old is seen in the emergency department for a fall. The client has bruises on the upper arms and appears depressed. The client is accompanied by a grandchild, who is unkempt, glassy -eyed, and whose breath smells of alcohol. Which of the following should be a priority with the nurse? A) Assess whether the older adult is safe in the home environment. B) Determine whether legal interventions are appropriate. C) Assess the client's degree of frailty and chronic health problems. D) Determine the mental capacity of the older adult

A The first priority should be to see whether the older adult is safe and then determine his competency. Legal intervention can be pursued after safety and competency are determined. The degree of frailty and chronic health problems is assessed with safety issues and determination of competency

A nurse identifies those who are at risk for familial stress. Which of the following persons exemplifies the sandwich generation? A) A 50 -year -old who balances the care of an 82 -year -old parent and a 20 -year -old child B) A 58 -year -old whose elderly parents have been forced to live in separate care facilities C) A 72 -year -old who deals with own health problems with the care of a grandchild D) An 83 -year -old who is the sole caregiver for the 79 -year -old spouse

A The increasing numbers of middle -aged adults who simultaneously juggle the demands of caring for older and younger generations are referred to as the sandwich generation.

A nurse verifies the health belief system of the Puerto Rican older adult client. Which of the following beliefs is this client most likely to hold? A) Health is a gift or reward given as a sign of God's blessing and goodwill. B) Health is obtainable by reaching a mature age. C) Health is the absence of disease. D) Health is the quality of wholeness associated with healthy functioning.

A The magico -religious perspective is common among Latino and Caribbean groups. Those with this perspective believe that health is a gift or reward given as a sign of God's blessing and goodwill. Scientific perspective holds that health is an absence of disease. Holism associates health with healthy functioning and well -being

A 75 -year -old woman who often used to go out to dinner with her friends has stopped from going out because she has been experiencing urinary incontinence and is afraid of having an "accident" in public. When her child asks her why she doesn't go out with her friends anymore, she says, "I'm getting too old for such foolishness." How can the nurse best assist this client? A) Assist the client to view this functional limitation as temporary and treatable. B) Encourage the client to accept this consequence of growing old. C) Rephrase the situation to one of control, and allow the client to make the decisions. D) Teach the client that majority of older adults rate their health as good to excellent.

A The majority of older adults do rate their health as good, but she doesn't and can't until her issue is reframed to one that she can control. Allowing her to make the decisions is good, but her current decision is to passively and inaccurately accept this treatable condition.

While a nurse is performing a recently admitted hospital client's morning care, the client states, "I'm pretty sure I'll never see my own apartment again." Which of the following responses by the nurse best demonstrates effective communication? A) "What is it that makes you feel that way?" B) ""I'm sure that's not going to be the case." C) "All in all, you're doing quite fine." D) "There's a lot that we can do, dear, to make sure that you do."

A The nurse demonstrates empathy and respect, while also facilitating further assessment findings around the client's beliefs for recovery through the use of an open -ended question. The nurse should not demonstrate a false reassurance of recovery, nor downplay the client's concerns. The nurse does not use patronizing terms of address (dear, honey, sweetie).

Which of the following interventions most closely aligns with the practices of health promotion? A) Leading a flexibility and mobility class among older adults B) Providing presurgical teaching to an older adult prior to hip replacement C) Administering an anti -inflammatory and analgesics to an older adult with osteoarthritis D) Teaching an older adult how to administer her inhaled bronchodilators independently

A The practice that best characterizes health promotion is facilitating exercise. Presurgical teaching, administering drugs, and teaching independence with medication are useful interventions, but they do not characterize the behavior changes of health promotion as well as an intervention such as an exercise class.

A nurse assesses an older adult's color perception. Which of the following colors should the nurse expect the client to have the most difficulty visualizing? A) Blue and violet hues B) Yellow tones C) White and off-white D) Tan and brown wavelengths

A These age -related changes decrease responsiveness of the lens and increase the diffusion of light rays, resulting in fewer rays reaching the retina. The most detrimental effect occurs with the shorter blue and violet wavelengths.

A nurse assesses the risk of the members of the community. Which of the following are most likely to be living at or below the poverty line? A) An 83 -year -old single woman B) A couple who are both 72 years old C) A white 73 -year -old man D) A Hispanic couple in their 60s

A Women and those over the age of 75 are more likely to live in poverty. Couples and those 65 to 74 are less likely. Five percent of older white men live in poverty

A nurse determines risk factors for an 81 -year -old client's plan of care. Which of the following characteristics of the client would the nurse consider as a risk factor? (Select all that apply.) A) Chronic bronchitis B) Loss of bone density C) Decreased vital lung capacity D) Delayed gastric emptying E) Digoxin (Lanoxin) toxicity

A,E Chronic bronchitis would be considered a pathologic process and risk factor for disease, rather than an expected or inevitable age -related change. Adverse medication effects are also considered risk factors. Loss of bone density, decreased vital lung capacity, and delayed gastric emptying are all examples of normal, age -related changes.

A nurse performs an assessment on a 93 -year -old client. Which of the following assessment findings are age -related changes? (Select all that apply.) A) Ectropion B) Enophthalmos C) Erythematosus D) Eschar E) Exophthalmos

A,B Enophthalmos is the appearance of sunken eyes and ectropion is when the lower eyelid falls away from conjunctiva, causing decreased lubrication, both of which are age -related changes. Erythematosus refers to the presence of erythema, eschar is the necrotic scab on a wound, and exophthalmos is the protrusion of the eye.

A nurse in the long -term care facility plans a meeting to assist an older adult and family discuss end -of-life care options. Which of the following interventions is appropriate for the nurse to include in preparation for this event? (Select all that apply. ) A) Assist the older adult to his or her wear hearing aid. B) Assure that the older adult is well rested. C) Obtain a private meeting room. D) Premedicate the older adult with Ativan (lorazepam). E) Schedule the meeting after a meal.

A,B,C An important role of nurses is to promote optimal decision -making capacity by identifying and addressing the factors that influence cognitive functioning and is within the realm of nursing responsibilities (such as wearing hearing aid and being rested). Benzodiazepines and overstimulation (such as after a meal) can interfere with the older adult's capacity

A nurse assesses older adults in their own home. Which of the following questions are appropriate to include in this assessment of the bathroom? (Select all that apply.) A) Can the person enter and exit the tub safely? B) Does the color of the toilet seat contrast with surrounding colors? C) Does the tub have skid -proof strips or a rubber mat in the bottom? D) Is the height of the toilet seat appropriate? E) Is there a lock for the bathroom door?

A,B,C,D Can the person enter and exit the tub safely? Does the color of the toilet seat contrast with surrounding colors? Does the tub have skid -proof strips or a rubber mat in the bottom? and Is the height of the toilet seat appropriate? are all appropriate questions to ask when assessing the safety; the door lock is not helpful.

A nurse develops a plan to addressing dementia -related behaviors in an older adult with dementia. Which of the following interventions should be included in this plan? (Select all that apply.) A) Maintain a clutter -free environment. B) Implement regular rest periods. C) Place pictures of familiar people in very visible places. D) Lay out clothing in the order in which the items are to be donned. E) Test the client's memory with each conversation.

A,B,C,D Implement regular rest periods to compensate for fatigue and loss of reserve energy. Use simple pictures and place pictures of familiar individuals in visible areas. Keep the environment free of clutter and place dangerous substances in an inaccessible area. Avoid persistent testing of memory

A nurse monitors for depression in the older adult population. Which of the following are a risk factor and a functional consequence of depression in the older adult? (Select all that apply.) A) Chronic pain B) Functional impairment C) Hypernatremia D) Nutritional deficiencies E) Renal impairment

A,B,D Chronic pain, functional impairment, and nutritional deficiencies are both contributing factors and consequences of depression in the older adult. Renal impairment and hypernatremia are not specifically related to depression

A nurse who regularly visits an adult daycare center has noted evidence of a hearing deficit in a man who has no documented history of hearing loss. Which of the following factors should the nurse consider when attempting to ascertain the etiology of the man's hearing loss? (Select all that apply.) A) Genetic factors B) Environmental conditions C) Fluid and electrolyte imbalances D) Ototoxic medications E) Atherosclerosis or thrombotic events

A,B,D Medications, genetic factors, and environmental factors are all among the many potential contributors to hearing loss in older adults. Fluid and electrolyte imbalances, atherosclerosis, and thrombosis are not commonly implicated in hearing loss among older adults

A nurse is identifying positive functional consequences as part of the development of an older client's care plan. Which of the following outcomes exemplifies the concept of positive functional consequences for an older adult? (Select all that apply.) A) The older adult with arthritis can walk 1 mile without pain. B) The older adult who is overweight develops a plan to lose 2 lb a month. C) The older adult has constipation from pain medication. D) The older adult schedules cataract surgery

A,B,D Positive functional consequences can result from automatic actions or purposeful interventions. Older adults bring about positive functional consequences (also called wellness outcomes) when they compensate for age -related changes and risk factors, such as cataracts and chronic conditions. Nurses help older adults achieve positive functional consequences by teaching about health promotion interventions to improve functioning and quality of life.

A nurse is beginning a new job in an area with a large African American population. Which of the following statements will assist the nurse to understand this ethnic/race culture to better plan nursing care? (Select all that apply.) A) African Americans as a group have a wide range of socioeconomic conditions. B) Female -headed households are common among African Americans. C) Lifestyle and risk factors account for the health disparities with older African Americans. D) Older African Americans are more likely than other older Americans to live alone. E) Older African Americans are more likely to be caring for their grandchildren.

A,B,E Female -headed homes are a common family structure among African Americans. The differences in health outcomes between African Americans and other groups are attributed mostly to disparities in health care provision. Older African Americans are more likely to live with family. Half of those older African Americans who live with their grandchildren are the primary care provider to those children.

A nurse assesses a client admitted to the subacute care unit. The client is weak and underweight. Which of the following laboratory abnormalities would be related to undernutrition in this client? (Select all that apply.) A) Low albumin B) High hematocrit hemoglobin ratio C) Low serum iron and ferritin levels D) Decreased platelet count E) Elevated sedimentation rate

A,C Serum iron, iron -binding capacity, ferritin, and albumin indicate undernutrition. The other laboratory values do not

Which of the following methods can be used to informally assess an older adult's visual skills? (Select all that apply.) A) Ask the person to look out a window and describe certain details. B) Perform a standard confrontation test to assess central vision. C) Place good illumination and ask the person to read printed material with various type sizes. D) Perform a standard vision test, testing each eye separately and allowing the person to cover the other eye with a hand.

A,C, Nurses informally assess vision by asking the older adult to read printed material with various type sizes and describing details of a scene at a distance. A standard confrontation test is a gross measurement of peripheral vision fields. With standard vision tests, each eye is tested separately, and one should avoid using the hand as a cover

Which of the following actions exemplifies the nurses' role in home care of an older adult? (Select all that apply.) A) Coordinate a multidisciplinary team. B) Perform ADL care for clients. C) Provide resources to caregivers to reduce caregiver stress. D) Refer available community resources. E) Teach about interventions to provide quality care.

A,C,D,E Nurses who provide skilled home care services typically assume a primary coordinating role with a multidisciplinary team. Nursing responsibilities include referrals for additional services. Nurses direct their interventions toward the caregivers providing teaching about interventions, and they address needs of the caregiver related to information about resources and ways to reduce caregiver stress

A nurse is conducting a medication assessment of an older adult client who will soon be receiving home care. Which of the following questions should the nurse include in this assessment? (Select all that apply.) A) "Are you a smoker?" B) "What is your typical diet?" C) "What over -the -counter drugs do you use?" D) "Do you use any herbs or dietary supplements?" E) "Do you drink alcohol?"

A,C,D,E Question relating to smoking, alcohol use, over -the -counter drugs, and herbs and dietary supplements should be included in a medication assessment. A client's diet, however, is not a common focus during a medication assessment

An older adult has developed hallucinations. For which of the following should the nurse assess? (Select all that apply.) A) Digoxin toxicity B) Hyperglycemia C) Infection D) Myocardial infarction E) Stroke

A,C,E Infection, digoxin toxicity, and a stroke can all lead to hallucinations. Hyperglycemia and myocardial infarction generally do not.

A nurse teaches an older adult about the antidepressant medication recently prescribed. Which of the following should the nurse include in the teaching? (Select all that apply.) A) Antidepressants can interact with alcohol and over -the -counter medications. B) Depression is uncommon in the older adult population. C) Expect adverse effects of the medicine; stop medication if they occur. D) Don't expect immediate improvement; a fair trial may take up to 12 weeks. E) The medication is to be taken only as needed

A,D Immediate improvement will not be evident, but a fair trial must be given to the medication as long as serious adverse effects are not noticed. The fair trial may take as long as 12 weeks, but some positive effects should be noticed within 2 to 4 weeks. Antidepressants can interact with alcohol, nicotine, and other medications, including over -the -counter medications, possibly altering the effects of the medication or increasing the potential for adverse effects. Depression is common in older adults, it's treatable, if the client sticks to the treatment plan and takes medication daily (not PRN). Medication should not be stopped without consulting the primary health care provider.

A nurse cares for an older adult at risk for a venous stasis ulcer. Which interventions should the nurse include in the teaching with this older adult? (Select all that apply.) A) Health promotion interventions B) Functional consequences C) Wellness outcomes D) Environmental modifications

A,D The nurse should include health promotion interventions and environmental modifications to create wellness outcomes for this client. Functional consequences and wellness outcomes are not interventions

After an older adult has had irrigation for removing impacted cerumen, which of the following interventions would be helpful for preventing a recurrence? (Select all that apply.) A) Ceruminolytic drops as indicated B) Cotton -tipped swabs daily C) Ear candling monthly D) Home oral jet irrigator bimonthly E) Examination by the health care provider every 6 to 12 months

A,E Prophylactic use of ceruminolytic agents can reduce the risk of impacted cerumen. Those at high risk for impaction should get an examination by qualified health care provider every 6 to 12 months. Teach all older adults to avoid putting anything smaller than their elbow in their ear: this includes candles, oral jets, and swabs. All of these are potentially harmful to the ear

Even in a high acuity situation, a nurse can assess the spiritual needs of a client. Which of the following questions is appropriate for the older adult admitted to the intensive care unit for sepsis? A) "Do you attend church services?" B) "Is there a spiritual leader we can call for you?" C) "What are your beliefs about death?" D) "What religion are you?"

B Asking about religion is less important than asking about spirituality, and while the nurse in the intensive care unit may not be able to complete a full spirituality assessment, they can obtain support for the client. Asking about death beliefs may be appropriate in a full spiritual assessment, but not for the high acuity client.

Assessment of an older adult's ADLs addresses parameters such as mobility, dressing, and elimination. In addition to these, which of the following categories should the functional assessment also include? A) Pain B) Mental status C) Previous medical history D) Integumentary assessment

B A brief mental status assessment is included on the ADL form. Including mental status in the functional assessment rather than using a separate mental status assessment tool reinforces the fact that cognitive function is an integral component of ADLs. In addition, it helps to determine whether ADL impairments are attributable, at least in part, to cognitive impairments, rather than primarily to physical limitations. Pain, medical history, and integument are all relevant assessment parameters, but none is explicitly included in a functional assessment.

A nurse cares for an 81 -year -old client whose current hospital admission has been prompted by an exacerbation of chronic renal failure. Which of the following actions by the nurse will best emphasize the goal of client wellness? A) Ask for the client's code status be changed to "do not resuscitate." B) Explore the client's abilities and strengths. C) Show the client others who are more ill. D) Teach the client that health problems do not have to affect daily routines

B A focus on existing strengths and abilities can foster wellness in older adults, even when they are experiencing health challenges. It is inappropriate to actively compare clients with each other, and changing Mr. Say's code status is unlikely to promote wellness, even if this is necessary. It is inaccurate to claim that health problems do not affect daily routines

A nurse initiates an acute care for elders unit in a medical facility. Which of the following factors should the nurse include when teaching the nursing staff about delivering medications to the older adults on the unit? A) Drug metabolism shifts from the liver to the kidneys as individuals age. B) Older adults face an increased risk of adverse medication effects. C) Older adults tend to achieve clearance of medication faster than do younger patients. D) Older adults tend to need more frequent doses of a drug to achieve therapeutic effect

B Age -related physiologic changes result in an overall increased risk of adverse drug effects. Drug clearance tends to be slower, so lower doses are normally required. There is no change in which organs are responsible for metabolism of drugs in older adults .

A nurse manager of a nursing care facility reviews potential health and safety interventions and outcomes with the nursing staff. Which of the following interventions places emphasis on quality of life with the best possible health and safety outcomes? A) Telling the resident about his or her schedule for the day B) Allowing the resident with a history of falls to walk with the help of an assistant C) Discouraging the resident who states they are tired from participating in activities D) Advising the resident not to attend church because the resident is not Catholic

B Allowing the resident with a history of falls to walk with the help of an assistant places emphasis on quality of life with the best possible health and safety outcomes. Telling the resident about his or her schedule for the day does not address safety or autonomy. It is not quality of life to discourage the resident from participating in activities (even if tired). Advising the resident not to attend church because the resident is not Catholic is unethical

A home health nurse performs an admission on a 90 -year -old client who has a small dog. The client states that the most important problem is urinary incontinence. The client's blood pressure is 135/90 mm Hg, with last bowel movement 3 days ago. Which of the following areas addresses the person's goals? A) Safety B) Incontinence C) Blood pressure D) Constipation

B Although nurses address safety, blood pressure, and constipation as part of a comprehensive care plan, it is imperative to begin by addressing the older adult's priority.

A nurse teaches an older adult man to perform pelvic floor muscle exercises (PFME)? Which of the following should be included in a nurse's instructions? A) Interrupt the flow of urine several times each time you urinate. B) Identify the correct muscle by making the base of your penis move up and down. C) Contract your legs and buttocks while contracting the pubococcygeal muscle. D) Perform the exercise while standing over the toilet

B An important element of teaching about pelvic floor exercises is to identify the pubococcygeal muscle and practice contracting and relaxing this muscle. For men, this can be done by raising the base of the penis. Once the muscles have been identified, do not continue to stop urinary flow. These exercises can be performed sitting, standing, or lying down. Keep legs, buttocks, and abdomen relaxed.

A nurse reviews the medication list of an older adult upon transfer from the hospital to an extended care facility. Which of the following methods is most likely to reduce the occurrence of adverse effects? A) Administer medications at the same time every day with meals. B) Compare the list to the Beers criteria list and notify the health care provider of any on the list. C) Request that the client's medications be put on hold and restarted one at a time. D) Stop the administration of GI and narcotic pain medications.

B An important theme of the Beers criteria and other guidelines is that medications are determined to be appropriate or inappropriate in relation to the patient's condition. Some medications should be given with meals, while others should not. There is no need to stop all medications at this time; nor should the GI and pain medications be stopped

A nurse is discussing an older adult's recent diagnosis of rheumatoid arthritis with a colleague. Which of the nurse's statements reflects an accurate view of the relationship between aging and wellness? A) "It's important that the individual knows this is an expected part of growing older." B) "We need to teach the older adult how he can keep living a fruitful life in spite of his diagnosis." C) "We need to make sure our teaching is not too detailed for someone of his age." D) "We need to ensure his expectations of continuing to live alone are realistic.

B Answer B reflects a desire to foster a fulfilling and productive life for the older adult despite his diagnosis, and reflects an understanding of wellness and aging. Option A implies that illness is an inevitability of aging. Option C suggests that older adults have a limited capacity to learn by virtue of their age, and option D implies pessimism about the health and functioning of the older adult

A nurse assesses a 79 -year -old adult noting the presence of a white ring around the iris bilaterally. What is the correct term for this? A) Glaucoma B) Arcus senilis C) Arthritis D) Presbyopia

B Arcus senilis (also called corneal arcus) is evidenced by the development of yellow or gray -white ring around the iris. Glaucoma is related to pressure changes, not changes to the iris. Arthritis is unrelated to eye health. Presbyopia is an age -related visual acuity change.

A nurse assesses an older adult's abstract thinking ability. Which of the following questions is most appropriate? A) "Do you know why you are in hospital right now?" B) "What do a dog and a cat have in common?" C) "What goals do you have for your treatment and recovery?" D) "What would you do if you found a stamped, addressed letter on the ground?"

B Asking what traits two similar, but not identical, objects share is a way of gaining insight into a client's ability to think abstractly. Option A addresses insight, not abstract thinking, while Option C is a useful assessment question, but not one that addresses abstract thinking. Option D could be used to assess the client's judgment

A 78 -year -old was diagnosed with colorectal cancer 18 months ago and underwent a round of chemotherapy. The most recent computed tomographic scan, however, reveals that the cancer has metastasized to the lungs and liver. The older adult states, "I feel quite well and do not wish to undergo another round of chemotherapy. " The client's children are adamantly opposed to their parent's decision to forgo treatment and have appealed to the nurse. Which factor is the priority consideration for the nurse to determine the best course of action? A) The client's prognosis B) The client's autonomy C) The family's wishes D) The client's treatment option

B Autonomy is highly valued in Western societies, and personal autonomy supersedes family wishes and the medical facts about a client or client's situation.

As part of a comprehensive physical assessment of an older adult client, a nurse is performing an otoscopic examination. Which of the following assessment findings most clearly warrants further assessment and possible intervention? A) There is a small amount of cerumen visible in the ear canal. B) The epithelial lining is bright red. C) The tympanic membrane is intact. D) The tympanic membrane is a pearl -gray color.

B Bright red epithelial lining in the ear is an abnormal finding; the lining should be pinkish white. A small amount of cerumen and an intact, pearl -gray tympanic membrane are expected findings.

A nurse who provides care in a clinic comes into contact with numerous older adults, many of whom have bruises of various sizes and stages on their body. What pattern of bruising is most suggestive of possible abuse? A) Significant bruising on the shin region of a client's leg B) Bruising on both ears and both sides of the neck C) Bruising on the back of a client's hands D) Bruising on both of a client's elbows

B Bruising on the neck and ears is not typically accidental. Conversely, bruising on the backs of the hands, elbows, and shins is more common and less likely to raise the suspicion of abuse.

A nurse's colleague states, "Older people who live in the country are a lot healthier than city folk." Which statement by the nurse is most appropriate? A) "The differences aren't large, but rural adults do have better health outcomes than do city dwellers." B) "But chronic conditions are more common among rural adults." C) "Overall, yes. Higher levels of family support translate into longer average life spans for rural adults." D) "Unfortunately, no. And this is mostly attributable to the problem of homelessness.

B Chronic conditions are overrepresented among rural adults. Overall, rural adults have worse health outcomes than do urban dwellers. While family support levels are generally high, this does not mean that rural adults live longer lives on average. Lack of access and other factors, rather than homelessness, are cited as reasons for the differences in health outcomes between rural and urban adults.

A nurse determines that a client does not remember current events and has difficulty using technology. The nurse should consider that the client may have difficulty with which of the following? A) Participating in reminiscence group B) Digitally recording blood glucose monitor C) Remembering to weigh daily D) Understanding when to notify health care provider

B Contextual theories and everyday problem solving emphasize that older adults are able to remember affective and personally relevant information. The client may need to record the blood glucose on paper.

A nurse has recently begun to provide care to older adults in a large, urban hospital. Having lived until recently in an ethnically homogeneous region, the nurse has begun to recognize the significant differences in priorities and perspectives of clients from other cultural groups and has taken action to learn about these groups. What stage of cultural self-assessment is this nurse demonstrating? A) Unconsciously incompetent B) Consciously incompetent C) Consciously competent D) Unconsciously competent

B Cultural competence begins with unconscious incompetence as a state of not being aware that one is lacking knowledge about another culture. When the person becomes aware of this knowledge gap, he or she progresses to a state of conscious incompetence and takes actions to learn about the cultural group; this stage is demonstrated by the nurse in this question. A person progresses to a stage of conscious competence by verifying generalizations and incorporating culture -specific interventions in care. The final stage is unconscious competence, when knowledge of the cultural group is fully integrated into one's thinking and approach.

A nurse plans the diet for an older adult with congestive heart failure. Which of these nursing interventions would be most successful to encourage optimal nutrition? A) Encourage calorie supplements. B) Provide 55% of calories from complex carbohydrates . C) Teach older adults to sit upright for 2 hours after a meal. D) Use moderate to large amounts of flavor enhancers.

B Dietary guidelines for older adults recommend a daily intake of five to nine servings of fruits and vegetables; 55% of calories need to come from complex carbohydrates. Older adults need fewer calories with increased quality of nutrients in their nutritional requirements. Older adults with presbyesophagus must sit upright for 30 minutes to 1 hour after eating. Flavor enhancers (except lemon) contain sodium and need to be used in small amounts for older adults who have a diminished sense of taste.

A nurse in a Medicare - and Medicaid -funded nursing home performs assessments and develops care plans. Which of these statements is true of the functional assessments the nurse is likely to perform? A) The nurse will address core ADLs but not more complex IADLs. B) The nurse will identify changes in the older adult's function over time. C) The nurse will utilize various functional assessment models. D) The main goal of functional assessments will be to ensure older adult safety.

B Functional assessments consider an older adult's functional status and changes in this status over time. They include both core ADLs and more complex IADLs. The nurse is likely to use the Minimum Data Set for Resident Assessment and Care Screening, as mandated for Medicare - and Medicaid -funded facilities. While safety is a consideration in functional assessment, the main goal is determining the older adult's need for assistance and for planning care

A nurse interviews a client who is 82 years old and has several chronic conditions, including type 2 diabetes and heart failure. The client expresses feeling of more satisfaction with life now than when younger. Which phenomenon is the client expressing? A) Metamemory B) The paradox of well -being C) Crystallized intelligence D) Neuroplasticity

B Gerontologists have identified a paradox of well -being among older adults, which describes the phenomenon of older adults suffering significant losses of health, cognition, and social functioning but reporting high levels of well -being and positive emotions. Metamemory, crystallized intelligence, and neuroplasticity are phenomena that are not directly related to subjective well -being and satisfaction.

A nurse plans interventions to promote wellness in older adults. Which of the following interventions is most appropriate to meet this goal? A) Talking with the physician about available treatment options for an older adult with an acute illness B) Facilitating early mobilization to prevent muscle wasting and loss of function in an older hospital client C) Deferring the final decision regarding an older adult's choice of assisted living facility to the person's son and daughter D) Placing a 76 -year -old on the waitlist for a kidney transplant

B Goals of the Functional Consequences Theory include improving or preventing declines in functioning and addressing quality -of-life concerns. Discussing treatment options, having family members make an older adult's decisions, and placing an individual on a waitlist for a transplant are not direct manifestations of this principle

A nurse assesses a 61 -year -old adult who reveals that he can't process as quickly as when younger, and that "all these people talk about multi -tasking, but I can't do that!" Which of the following responses by the nurse is appropriate? A) "Have you had any other symptoms of cognitive impairment?" B) "Slower processing of information is an age -related change, and there are things you can do to help with this." C) "The declines in cognitive skills usually begin around the age or 60." D) "You shouldn't expect to see a decline the cognitive functions that you use all the time."

B Healthy older adults will not experience any significant cognitive impairment that interferes with daily life, but they will notice minor deficits in some aspects of cognitive function and improvements in other aspects. The earliest cognitive changes are due to decreased perceptual speed. The other distracters do not answer his question. It is important for the nurse to address the client's concerns; in this case, the client is asking if it is expected to already have age -related functional consequences. Ag e -related declines in some cognitive skills begin around the age of 40, but there are substantial individual variations in these changes. Cognitive functions that depend on experience, accumulated knowledge, and well -practiced tasks (e.g., vocabulary) do not decline in healthy older adults, and may even improve.

An older adult started an antidepressant 1 week ago. The client states, "I don't want to take that pill, it's not doing anything." Which of the following responses by the nurse is most appropriate? A) "That is fine, it is your right to refuse medications. " B) "It is too soon to see effects; positive effects may begin around 3 weeks." C) "Let's notify the primary health care provider to try another type of medication." D) "What side effects are you having?"

B Immediate improvement will not be evident, but a fair trial must be given to the medication as long as serious adverse effects are not noticed. The fair trial may take as long as 12 weeks, but some positive effects should be noticed within 2 to 4 weeks. If one type of antidepressant is not effective, another type may be effective. The right to refusal is the seventh medication right; however, it is the nurses' responsibility to ensure that the client is informed before accepting the refusal.

Despite the fact that older adults are proportionately the highest users of health care services, many nurses harbor misconceptions and deficits in practice related to gerontological nursing. What is the most likely solution to this problem? A) A shift from the treatment of older adults in institutional settings to home care B) Increased nursing education and clinical experience specific to working with older adults C) A focus on early discharge planning for older adults in hospital settings D ) Increased use of aggressive pharmacologic interventions in the treatment of acute illnesses in older adult

B Individual gerontological nurses as well as national nursing bodies have joined in a call for increased education to better enable nurses to meet the diverse health needs of older adults. Early discharge planning, increased home care, and aggressive drug treatment are measures that may be appropriate in certain contexts, but none is likely to improve nursing care of older adults at a broad level.

A 69 -year -old woman is saddened by her recent diagnosis of type 2 diabetes, which is a stressor that will make numerous demands on her life in the coming years. Which of the following actions demonstrates a problem -focused approach to this stressor? A) Eliciting support and sympathy from her sister and neighbor B) Obtaining diabetic cookbooks and learning to change her cooking habits C) Seeking out a second opinion from another physician D) Deciding to make no lifestyle changes despite her new diagnosis

B Making tangible changes to address a problem, such as changing lifestyle to accommodate a new diagnosis, demonstrates a problem -focused approach to coping. Seeking support is emotion focused and seeking a second opinion is not necessarily a solution

An older adult has been accompanied by an adult child to visit a primary care provider. The child has expressed concern about the client's increasing apathy, isolation, and apparent sadness over the past several months; and the client acknowledges many of the symptoms of depression. Which of the following assessments should the nurse prioritize? A) Functional assessment B) Medication assessment C) Musculoskeletal assessment D) Cardiovascular assessment

B Medications may be risk factors for depression in numerous ways. A functional assessment is necessary, but this is more likely to ascertain the effects, rather than causes, of her depression. Musculoskeletal and cardiovascular assessments are secondary.

Which of the following points should the nurse emphasize when educating older adults about memory and cognition? A) Long -term memory loss is normal. B) Using calendars, notes, and imagery can help enhance memory. C) Drinking caffeinated beverages for mental stimulation is a good idea. D) Having a diminished capacity for learning is an inevitable part of growing older .

B Metacognition means that an individual understands his or her own cognitive process, and this process will impact performance. Health education provides information about techniques to enhance cognitive abilities. Older adults benefit from internal and external memory -enhancing techniques, such as calendars, imagery, and notes.

An 84 -year -old client has been living in an assisted living facility for several years but is now faced with the prospect of relocating to a nursing home. Which of the following characteristics of the client's current situation is most likely to prompt this move? A) The development of a severe, acute health problem B) A decrease in the client's level of function and activities of daily living (ADLs) C) Exacerbation of a chronic health problem that may require medical treatment D) A change in the level of the client's social support

B Nursing home settings are becoming increasingly diverse, but a common feature of older adults who are admitted to nursing homes is a decrease in function and ADLs. Acute health problems that require medical treatment necessitate hospital admission, and a change in social support would not necessarily prompt a move from assisted living.

An older adult, aged 72, with type 2 diabetes and coronary artery disease is admitted to a long -term care facility. The client takes glipizide (Glucotrol) and isosorbide mononitrate (Imdur). The medical history states that the client drank 4 ounces of whiskey per day for many years. Which of the following actions should be a priority for the admitting nurse? A) Assess and observe for depression. B) Assess for hypoglycemia and hypotension. C) Evaluate the client for renal failure. D) Evaluate blood work for changes in electrolytes.

B Older adults are more susceptible to developing medication-alcohol interactions. Age -related changes in body composition can cause higher levels of alcohol to be absorbed into the bloodstream. Alcohol enhances vasodilation when an individual takes a nitrate, and there is potentiation of oral hypoglycemics by alcohol. CNS depression occurs when alcohol interacts with barbiturates and meprobamate, which this client is not taking. There is no need to evaluate for renal failure or changes in electrolytes; these are not known medication-alcohol interactions.

An older adult was diagnosed with depression shortly after relocating to the nursing home 6 weeks ago. What intervention should the nurse implement to address the depression? A) Teach the client about the problem of suicide in older adults. B) Provide opportunities for the client to engage with other residents. C) Direct the client to list all the positive aspects of her present circumstances. D) Appoint another resident as a "buddy" to accompany the client during the day.

B Social engagement and contact of all types has the potential to aid in the treatment of depression. Appointing a "buddy," however, is likely to be construed as intrusive and is unfair to the other resident. Asking an individual to focus on positives may be seen as simplistic. Teaching about suicide is unlikely to alleviate the signs and symptoms of depression

A nurse assesses a community of older adults. Which of the following persons is at highest risk for a shortened life expectancy? A) A college professor born in 1956 B) A homemaker born in 1957 C) A nuclear engineer born in 1958 D) A nurse born in 1959

B Socioeconomic characteristics that are most strongly correlated with healthy aging are poverty and lower educational level. Limited English proficiency and poor health literacy skills are two variables that have a negative impact on health and functioning. The college professor, engineer, and nurse positions require more education.

A nurse at a long -term care facility has completed the admission assessment of a 79 -year -old male resident. The resident has identified himself as gay and has expressed sadness at having to leave his partner of several decades in order to move to the facility. The nurse should recognize that this resident is likely to have a history of: A) Homelessness B) Stigmatization C) Nominal employment or unemployment D) Infectious diseases

B Stigma is a common experience among LGBT (lesbian, gay, bisexual, and transgender) older adults, and the likelihood of this is greater than the likelihood of homelessness, unemployment, or infectious diseases

A gerontological nurse presents at a local conference regarding recent findings in age -related hearing loss. Which age -related changes indicated in hearing loss and speech perception should the nurse include? A) Changes to the external auditory canal and pinna B) Degenerative changes to the auditory brainstem C) Growth of longer and thicker hair in the ear canal D) Thickening of the tympanic membrane

B Studies suggest that central auditory structures (e.g., primary auditory cortex and auditory brainstem) account for a significant component of hearing loss in older adults, particularly with regard to speech perception. The external ear changes that occur with normal aging are not directly related to hearing loss (including hair growth in men). The tympanic membrane thins and stiffens with age

A nurse is working with a 79 -year -old client newly diagnosed with osteoporosis. Which of the following interactions promotes achievement of wellness outcomes? A) The nurse performs strength and mobility training appropriate to the client's age and diagnosis. B) The nurse teaches the client about bone density in older women and the role of vitamin D and calcium intake. C) The nurse plans interventions in light of the body-mind-spirit interconnectedness of the client. D) The nurse teaches the client about how her risk factors are a consequence of age

B Teaching about bone density and the role of vitamin D reflects an acknowledgment that the client's diagnosis is attributable to both the client's age -related changes and risk factors such as inadequate nutrient and mineral intake. Strength and mobility training may well be an appropriate intervention, but it does not necessarily acknowledge the interplay of risk factors and age in the client's diagnosis. The body-mind-spirit interconnectedness is a component of the Functional Consequences Theory, but it does not address the relationship between risk factors and age per se. Teaching about risk factors as a consequence of age implies that these factors are an inevitable consequence of age, which is not the case.

An older adult is sore from "doing too much in the yard yesterday." Which statement by the nurse best promotes healthy aging? A) "It's time to start exercising and eating right." B) "Let's look at how we can improve your health so you can do more." C) "Of course you can't do as much as you did before, you need to pace yourself." D) "You need to act your age, and let others do that work."

B The Functional Consequences Theory for Promoting Wellness in Older Adults emphasizes those factors that nurses can address through health promotion interventions. The client self-determines, and the nurse teaches and guides. If health care providers hold the perspective of "what do you expect, you're old, " then reversible disease conditions may go untreated

A nurse differentiates between dementia and depression in an older adult. Which of the following assessment findings leads the nurse to believe that the client has depression? A) The client has socially unacceptable behaviors. B) The client is negativistic. C) The client's mood fluctuates. D) The client's mood is distractible

B The affect of depressed people is generally sad and negativistic and is not influenced by external circumstances. By contrast, the affect of people who have dementia fluctuates more and changes in response to distractions

A 70 -year -old client with urosepsis is admitted to a nursing unit. The labs include elevated sodium, blood urea nitrogen, hematocrit, and albumin. Which of the following nursing diagnoses is priority for this client? A) Constipation B) Fluid volume deficit C) Imbalanced nutrition: less than body requirements D) Impaired tissue perfusion

B The appropriate nursing diagnosis is fluid volume deficit. Blood values that may be altered in dehydration include elevations in sodium, hematocrit, creatinine, osmolality, and blood urea nitrogen. While the client may develop constipation, it is not the priority at this time. Albumin will be decreased with poor nutrition but increased with dehydration. An elevation in these labs does not implicate impaired tissue perfusion.

A nurse assesses older adults at a senior center. One older adult, age 78, has a body mass index (BMI) of 15. Which response by the nurse is appropriate? A) "You are too skinny." B) "Have you been losing weight?" C) "Have you tried to lose this extra weight?" D) "Congratulations your BMI is great."

B The nurse uses therapeutic communication to assess the weight loss. Unintentional weight loss is considered a significant indicator of poor nutrition. Healthy adult BMI is between 18 and 25 and may extend to 30 for older adults.

The children of a resident of a nursing home have approached the nurse because they believe their parent is being manipulated by a person who also lives in the facility. Their parent has a diagnosis of early -stage Alzheimer disease and various comorbidities that affect mobility and function. How should the care team appraise the parent's decision -making capacity? A) Her decision -making ability is nullified by the presence of a dementia. B) Her decision -making capacity should be determined according to objective criteria. C) She should be asked to demonstrate sound decision making in minor matter before being allowed to make more important decisions. D) A surrogate should be appointed to make her decisions because she has been diagnosed with Alzheimer disease

B The presence of dementia does not necessarily render a person incapable of all decision -making ability. Rather, this ability should be analyzed according to criteria of understanding, reasoning, choice, and expression.

A nurse assesses a 66 -year -old woman who strained a muscle. The client attends the gym daily, and states, "I injured my muscle grouting the floor tile getting ready for the bridge class I teach." Which of the following categorizes this client's aging? A) Healthy B) Active C) Productive D) Successful

B The scenario does not show social participation, nor does it address whether or not the client is fully aging well (successful aging). The client does show healthy aging, but active aging better fits the information presented (active physically and mentally).

After a scheduled trip to her optometrist, a 70 -year -old has been told that the pressure in her eye is high and she needs to be monitored and treated to prevent damage to the optic nerve. What is this person's diagnosis? A) Cataracts B) Glaucoma C) AMD D) Presbyopia

B The term glaucoma refers to a group of eye diseases in which the ganglion cells of the optic nerve are damaged by an abnormal buildup of aqueous humor in the eye. Increased intraocular pressure is not implicated in the development of cataracts, AMD, or presbyopia.

A home health nurse visits a client who has a history of alcoholism and dementia. The client's words are incoherent, and the client's clothes are filthy. The client is unsteady and leaning to the right, and the room in the rooming house is in disarray. The only word that the nurse can clearly identify is "no." Which action by the nurse is most appropriate at this time? A) Ask the neighbors what has been happening. B) Call emergency services for transport to a hospital. C) Leave and return later. D) Search the room for empty bottles

B Victims of self-neglect are likely to have the following characteristics: older age; chronic illness; functional limitations; solo living arrangements; social isolation; inadequate economic resources; and dementia, mental illness, substance abuse, or hoarding behaviors. If the elder is incapable of deciding whether to accept or reject emergency services, then these services should be provided.

A nurse teaches a client and care partner about cholinesterase inhibitors. Which of the following statements should the nurse include in the teaching? A) "Rivastigmine (Exelon) has a high chance of interacting with other medications." B) "Nausea, vomiting, diarrhea, and loss of appetite can be prevented or reduced by starting with a low dose." C) "Rivastigmine is only for treatment of mild Alzheimer's disease and will be discontinued as the disease progresses." D) "You should have a 'drug holiday' monthly to improve the medication's functioning."

B When administering medications to older adults, it is imperative to start with lower doses and increase the doses slowly. Exelon is less likely to interact with other drugs and may be safer and better tolerated in people. It will continue and other medications may be added. The effectiveness of cholinesterase inhibitors is diminished significantly if it is stopped and then restarted.

A nurse has observed an increasing number of older Asian Americans in the hospital. Which of the following statements regarding Asian cultures will best assist the nurse to plan nursing care? A) Asian Americans as a group have lower mortality rates. B) Health is often viewed as a state of physical and spiritual harmony. C) Older Asian Americans are more likely than other Americans to live alone. D) Care of elders is commonly provided in institutional environments such as nursing homes.

B While it is true that Asian Americans as a group have lower mortality rates at all ages, the statement that best assists the nurse to plan care of the Asian client is that a common view of health from the Asian perspective is that of harmony in the spiritual and physical contexts. Asian Americans are less likely to live alone. It is more common for older Asian Americans to live with family than in nursing homes.

A client was diagnosed 3 years ago with a cognitive impairment, a condition that worsened over the next several months and which culminated in his recent death. An autopsy revealed numerous infarcted brain regions resulting from vessel occlusions. This client most likely suffered from which type of dementia? A) Alzheimer disease B) Vascular dementia C) Lewy body dementia D) Frontotemporal degeneration

B While the four major types of dementia are not discrete or mutually exclusive, vascular dementia is characterized by pathophysiologic processes including infarctions from occlusion of blood vessels. This pathophysiology is not characteristic of Alzheimer disease, Lewy body dementia, or frontotemporal lobe dementia

A community health nurse presents a class on "Aging in America: Living the Dream." Which of the following should the nurse stress when discussing retirement? (Select all that apply.) A) Delaying retirement until unable to work can be beneficial. B) Factors such as health, friendship relationships, and resources influence the transition. C) Sometimes the adjustment is more difficult for the partner who has not been employed. D) The adjustment to retirement is best accomplished quickly and with finality. E) A strong work ethic assists in the adjustment to retirement

B,C Factors such as health, family and friendship relationships, and economic and social resources influence the transition. Sometimes the adjustment is more difficult for the partner who has not been employed. Work ethic in society can diminish the retirees' status, delay of full retirement can assist with the transition, and delay in retirement is best done in a gradual manner (changing to part time, becoming self-employed)

A nurse assesses an older adult using a mini -mental status examination. The client is very slow to respond to the questions. Which of the following conditions may be present and will require follow -up by the nurse? (Select all that apply.) A) Lack of education B) Dementia C) Depression D) Confabulation E) Concrete thinking

B,C The nurse assesses the amount of time and effort expended in answering questions. This is particularly important when trying to differentiate between dementia and depression. Lack of education and concrete thinking would not slow the client in responding to assessment tools such as the mini -mental. Confabulation is when the client creates information.

Which of the following are crucial when assessing visual function in an older adult? (Select all that apply.) A) Asking the older adult to read the fine print on a medicine bottle without a magnifying aid B) Asking the older adult whether he or she can drive without difficulties at night C) Observing the older adult functioning in his or her normal environment D) Observing the older adult while he or she is reading a newspaper without glasses

B,C The nurse should observe the older adult's usual pattern of activities. These observations are best made in the person's usual environment and address the person's ability to carry out activities. Older adults who report difficulty driving at night may have cataracts or other visual impairments

A nurse instructs a class of older adult women about Kegel exercises. In which of the following urinary conditions would Kegel exercises be effective? (Select all that apply.) A) Functional incontinence B) Pelvic organ prolapse C) Stress incontinence D) Urge incontinence E) Urinary retention

B,C,D Pelvic floor muscle exercise (PFME) is an evidence -based practice that is effective as a first -line intervention for men and women with stress, urge, and mixed incontinence and in women with pelvic organ prolapse. Functional impairments are a major risk factor for the development of incontinence because they can interfere with the ability to recognize and respond to the urge to void in a timely manner. Because older adults have a shorter interval between the perception of the urge to void and the actual need to empty the bladder, any delay in reaching an appropriate receptacle can result in incontinence

Which of the following sources might nurses use to improve their cultural competence? (Select all that apply.) A) Discuss cultural norms with clients' families. B) Explore the resources in Online Learning Activities. C) Read journals and other references. D) Utilize organizations listed at the end of chapters. E) Write teaching materials in prominent local languages

B,C,D Professional sources are suggested to improve nurses' cultural competence. Nurses are encouraged to read journals and other references and by exploring the resources in Online Learning Activities in this chapter. Health care professionals are encouraged to contact local organizations to obtain culturally specific information about groups that reside in their locale. Clients' families should not be the primary source of data regarding cultural norms; nurses should be knowledgeable about cultural groups. Nurses should remain open to families input to their loved ones.

A nurse works to protect vulnerable populations and reduce health disparities. Which of the following nursing actions work toward that goal? (Select all that apply.) A) The nurse acknowledges that the clients in subgroups will not change beliefs or actions. B) The nurse communicates a nonjudgmental attitude toward health system beliefs. C) The nurse incorporates clients' belief systems into the plan of care. D) The nurse asks the client how the care system can incorporate the clients' health beliefs. E) The nurse teaches each client about preventive care.

B,C,D,E Health promotion interventions, such as teaching about prevention and early detection of certain conditions, are particularly important when caring for older adults who are members of a minority group. Nurses communicate nonjudgmental attitudes and asking open -ended questions to elicit information about each person's life experiences and cultural influences. Nurses need to be aware of the health beliefs that influence their clients, so they can adapt their interventions accordingly.

A intensive care nurse cares for an 83 -year -old with sepsis. The client exhibits illogical thinking and agitation. Which intervention should the nurse implement? (Select all that apply.) A) Administer a benzodiazepine. B) Assess for pain. C) Assure a quiet, dark sleep time. D) Initiate fall prevention program. E) Post pictures of client's family in room.

B,C,D,E Older clients with infection and those in the ICU are at great risk for delirium. Sleep, rest, pain control, and familiar items are interventions to minimize delirium. The client is a greater risk for falls so a fall prevention program should be initiated. Benzodiazepines should be avoided

A nurse assists adults to prepare for the changes that often occur in late adulthood. Which of the following psychosocial consequences occur because of life events during that period? (Select all that apply.) A) A broadening of social networks B) Adjusting to relocation from home C) Adjustment to a lower income D) Adaptation to chronic illnesses E) Coming to terms with one's mortality

B,C,D,E The life events of late adulthood have profound psychosocial ramifications, including reduced income, acknowledgment of mortality, relocation from home, and chronic illness. Social networks typically shrink rather than expand.

An older adult is admitted to a geriatric unit in the hospital. The nurse in the emergency department states that the client is oriented to one only. Which of the following actions should the admitting nurse perform? (Select all that apply.) A) Have a conversation with the client about challenges to cognitive functioning. B) Orient the client to the room, unit, and plan of care. C) Post a calendar with dates crossed off. D) Reassess orientation status. E) Repeat orientation as needed

B,C,D,E The nurse understands that the older adult may take time to orient to new surroundings and may need to be given cues to assist them. Temporary memory impairment because of acute medical problems may impact the orientation questions. Asking the client about his or her subjective cognitive challenges may yield meaningful information, but this does not necessarily gauge orientation

An older adult is admitted to the hospital with weight loss and cognitive impairment. To assist in the diagnosis of major depressive disorder, for which of the following should the nurse assess? (Select all that apply.) A) Decreased deep tendon reflexes B) Loss of interest or pleasure C) Psychomotor agitation D) Respiratory difficulty E) Sleep disturbances

B,C,E Diagnostic criterion for major depression includes depressed mood and/or loss of interest or pleasure along with at least five of the following signs and symptoms: weight loss, appetite change, sleep disturbances, observable, psychomotor agitation or retardation (i.e., slowness), fatigue or loss of energy, feeling worthless or excessively guilty, cognitive impairment, and recurrent thoughts of death or suicide. It does not include deep tendon reflexes or respiratory difficulty

A nurse assesses a 91 -year -old client in long -term care healing from bilateral broken legs caused in a fall. Today, the client developed new onset confusion and combativeness. Which of the factors must the nurse investigate as a source of this mental status state? (Select all that apply.) A) Social separation B) Hyponatremia C) Medication interactions D) Positional pain E) Urinary tract infection

B,C,E Sodium level, medications, and urinary tract infections can each lead to confusion and combativeness. While pain and social separation may be associated with confusion, they are unlikely to be the root cause of these new onset issues

A nurse prepares a presentation regarding elder abuse and neglect. Which of the following types of abuse should the nurse include? (Select all that apply.) A) Alcohol (substance) B) Financial C) Mandatory D) Physical E) Psychological F) Sexual

B,D,E,F The National Center on Elder Abuse (2013a) recognizes seven major types of abuse: physical, sexual, emotional or psychological, neglect, abandonment, financial or material exploitation, and self-neglect.

An 81 -year -old adult suffered an ischemic stroke 6 days ago. The client has failed to regain consciousness since the event. The care team has approached the client's family to obtain their views on inserting a feeding tube. Which of the following documents will allow the family to make a decision on the parent's behalf? A) A do not resuscitate (DNR) order B) A living will C) A durable power of attorney for health care D) A will

C A durable power of attorney for health care is an advance directive that takes effect whenever someone cannot, for any reason, provide informed consent for health care treatment decisions. A will, a DNR order, or a living will do not confer this authority on the client's family member.

A nurse completes the admission assessment of an 84 -year -old client to the long -term care facility. Which assessment finding would direct the nurse to document a deficit in the client's ADLs? A) The client experiences chronic pain as a result of rheumatoid arthritis. B) The client is able to ambulate with a wheeled walker for 60 ft but then requires a rest break. C) The client is able to wash self but requires assistance entering and leaving the bathtub. D) The client is unable to explain the rationale for each of the prescribed medications.

C ADLs include activities such as bathing, dressing, mouth care, hair care, dietary intake, transfer mobility, ambulation, bed mobility, and bladder and bowel elimination. Ambulation using an assistive device does not normally constitute a deficit in mobility. Chronic pain and unfamiliarity with one's medication regimen are significant assessment findings, but neither constitutes an ADL deficit

A nurse manager of the long -term care facility develops plans to reduce nutritional deficits. Which of the following interventions is appropriate to include in the plan? A) Encourage residents to eat in their rooms to minimize distractions. B) Offer four to five small meals a day rather than three larger meals. C) Promote oral care for residents multiple times each day. D) Provide incentives for residents to eat all the food on their trays

C Adequate oral care is important in the promotion of adequate food intake, because it enhances chewing, eating, and swallowing. Eating alone is associated with lower caloric intake. Offering incentives may be construed as coercive or patronizing. Frequent, small meals may be necessitated by certain medical conditions, but this is not a recognized strategy for the promotion of nutrition among a larger group of older adults.

A nurse is teaching a family of an older adult about the role of adult day centers. Which of the statements by the family member indicates a need for further teaching? A) "The day center can give me respite." B) "The day center can improve our quality of life." C) "The day center can be a useful alternative to medical care." D) "The day center can contribute to an actual improvement in dementia symptoms."

C Adult day centers are a community -based resource providing food, supervision, and activity, but are not designed to provide acute medical care. They provide caregiver relief and have been linked to improved quality of life and decreased symptoms of dementia.

An older adult is brought to the community clinic by an adult child with the concern of increasingly frequent lapses in memory. Which assessment question is most likely to identify potential risk factors for impaired cognitive functioning? A) "What did your mother and father die of?" B) "What line of work were you in?" C) "What medications are you currently taking?" D) "Where are you currently living?"

C Adverse medication effects can have a profound influence on the cognitive functioning of older adults. Genetic, environmental, and occupation factors are potential risk factors, but medications are more commonly implicated.

A nurse presents at a conference regarding functional consequences related to urinary elimination. Which of the following statements should the nurse include? A) "Most older women will develop urinary incontinence by the age of 85." B) "Most older adults will experience hypertrophy and relaxation of muscles in the urinary tract and pelvic floor." C) "Excretion of penicillin and cimetidine are decreased in older adults." D) "Healthy older adults experience an increase in glomerular filtration rate.

C Age -related changes in kidney function can impact water -soluble medications that are highly dependent on the glomerular filtration rate. This would include digoxin, penicillin, aminoglycosides, and cimetidine

An older adult has recently begun to display unprecedented lapses in short -term memory. The nurse overhears a colleague reassuring the person by saying, "Try not to worry; it's just a part of growing older." The nurse recognizes that this is an example of what phenomenon? A) Multiple jeopardy B) Gerontophobia C) Age attribution D) Implicit ageism

C Age attribution is the act of attributing a problem to age rather than to a pathology, as in the colleague's statement. Multiple jeopardy is the compounding of discrimination based on factors such as race and gender, while gerontophobia is the fear or hatred of older people. Implicit ageism is the unrecognized, negative perception of older people

A gerontological nurse conducts an assessment of an older adult who has a history of depression. Assessment reveals that the client has been drinking up to two bottles of wine each day for the last several months. What should the nurse teach the client about alcohol use and depression? A) "If you choose to use alcohol to address your depression, it's best to limit it to four to five drinks each day." B) "We recommend that everyone over the age of 70 abstain from drinking alcohol." C) "Alcohol has been shown to contribute to depression and vice versa." D) "If you quit drinking, your depression will likely improve

C Alcohol and depression have a synergistic relationship: alcohol causes depression, depression leads to alcohol abuse, which, in turn, exacerbates the depression. Four or five drinks daily is excessive, but abstinence is not necessary for all older adults. Abstinence is not guaranteed to improve symptoms of depression.

A nurse is conducting a study on the needs and living situations of older adults in the community. Which of the following statements should the nurse take into account? A) A majority of older Americans will live in a nursing home at some point. B) More older men live alone than older women. C) Assisted living arrangements have become increasingly common. D) Most older Americans reside in some form of institutional arrangement.

C Assisted living arrangements have become an increasingly common alternative to nursing homes. The number of Americans living in nursing homes has recently decreased, not increased, and more older women live alone than do older men. Ninety percent of older Americans live in an apartment or house, rather than in an institutional arrangement.

A nurse is aware that many health care providers prioritize the role of biology in the aging process. What is a weakness of biologic theories of aging as it applies to nursing? A) Biologic theories do not account for the differences in life expectancy between men and women. B) Biologic theories are unable to explain the role of cell division in life expectancy and the aging process. C) Biologic theories of aging do not adequately address issues of wellness and quality of life. D) Biologic theories of aging are unable to explain the significant increases in life expectancy that occurred in the 20th century.

C Biologic theories of aging do not address the significant influence of nursing, medical, and psychosocial interventions that can improve a person's functioning and life expectancy. More broadly, they do not address holistic questions surrounding wellness. They are generally able to account for increases in life expectancy and phenomena such as cell division

A new nursing assistant asks the nurse how best to approach a hearing -impaired older adult. Which of the following approaches should the nurse recommend? A) Raise the volume of your voice. B) Leave the radio on to calm the older adult. C) Lower the tone of your voice. D) Use exaggerated lip movements.

C Communication interventions for the hearing impaired should aim at clarity of words. This is accomplished by slowing the rate of speech and eliminating environmental noise and distractions. When communicating, lower the tone while speaking in a moderately loud voice

An 87 -year -old woman has a history of depression and hypothyroidism. She was recently diagnosed as having breast cancer. Her daughter tells her health care provider that her mother cannot participate in decision making about her care because she is too old. Which of the following statements is true about decision -making capacity? A) Determination of decision -making capacity is based on the older adult's diagnosis and chronologic age. B) The older adult has decision -making capacity if she understands most of the risks and benefits of medical treatment. C) The older adult needs to understand the issues involved in decision making and communicate about them. D) Decision -making capacity of older adults is always determined by a mental health judge.

C Decision making should not be determined by the diagnosis or the age of the older adult. The older adult needs to understand all of the risks and benefits of medical treatment. Decision -making capacity is determined by the health care practitioner or by the interdisciplinary team assigned to the client

A 78 -year -old client states, "I often have dry eyes, it is bothersome and irritating." What intervention should the nurse recommend? A) Daily rinses with tap water B) A medication vacation to determine if medications are the cause C) Use of over -the -counter artificial tears D) Keeping eyes closed for 3 to 5 minutes each hour

C For most older adults with dry eyes, the use of over -the -counter artificial tears or ocular lubricants usually will relieve symptoms. It would be inappropriate for the nurse to independently recommend a cessation of medication. Rinsing with tap water and keeping the eyes closed are not recommended interventions.

A nurse is explaining to a new colleague the similarities and differences between gerontology and geriatrics. Which of the following situations would most likely be addressed by a geriatrician rather than a gerontologist? A) Teaching older adults techniques to identify and deal with age -related changes B) Organizing and leading exercise classes to facilitate mobility C) Identifying and treating a client's vascular dementia D) Rearranging an older adult's apartment to minimize the risk of falls

C Geriatricians are medical specialists concerned with the treatment of diseases and disabilities common among older adults. The focus of gerontologists is a multidisciplinary approach aimed at emphasizing healthy and successful aging. Options A, B, and D fall under the auspices of gerontology.

A nurse who works with older adults is teaching a colleague about the similarities and differences between gerontology and geriatrics. Which of the following questions best conveys the focus of gerontology? A) "How can we secure more funding for research and development of drugs specifically for older adults?" B) "How can we teach older adults about the relationship between their lifestyle and their health?" C) "How can we help older adults maintain wellness as they age?" D) "How can we reduce the incidence of falls among older adults who live in care facilities?"

C Gerontology is the study of aging and older adults, and the focus of the discipline has shifted in recent decades to an emphasis on wellness and healthy, successful aging. As such, a focus on promoting and maintaining wellness best exemplifies the discipline. Geriatrics is associated with the diseases and disabilities of old people, and geriatric medicine is a subspecialty of internal medicine or family practice that focuses on the medical problems of older people. Origin: Chapter 5 - Gerontological Nursi

A nurse at the aging center organizes exercise classes including tai chi. Which of the following principles is the nurse incorporating? A) Disease prevention B) Environmental modification C) Health promotion D) Spiritual awakening

C Health promotion also emphasizes personal responsibility for health and self-care actions to achieve high -level wellness. Health is defined as the ability to function at the highest capacity despite the presence of age -related changes and risk factors. Environmental modifications are health promotion activities when they reduce risks or improve a person's level of functioning. Health promotion programs currently include both the traditional focus on disease prevention and health maintenance and on personal responsibility for health self-care actions to achieve high -level wellness

A nurse providing care in a long -term care setting is aware that the cognitive function of older adults does not necessarily decline on an inevitable trajectory. Which action has the greatest potential to enhance the cognitive function of residents and prevent cognitive declines? A) Encourage older adults to openly express their emotions and opinions. B) Provide residents with four or five low -fat, high -protein meals during the day. C) Encourage older adults to participate in mentally stimulating activities. D) Present older adults with numerous opportunities to make autonomous decisions

C Healthy diet, decisional autonomy, and emotional expression are all potentially beneficial, but participation in mentally challenging and stimulating activities has the greatest potential to protect and enhance cognition.

A 77 -year -old client was put on broad -spectrum antibiotics when hospitalized for sepsis. The client has a history of rheumatoid arthritis and a recurring problem with pneumonia. Which of the following theories best explains why the client has had these issues? A) Free radical theory B) Genetic theory C) Immunity theory D) Wear -and -tear theory

C Immunity theories focus on immunosenescence. Older adults are more susceptible to cancer, autoimmune disorders, and infections, a phenomenon that is known as immunosenescence. Wearing out is exacerbated by harmful factors, such as stress, disease, smoking, poor diet, and alcohol abuse. Free radicals are waste products of metabolism and they can damage cells. Current studies indicate that the genetic effect on longevity is due to modest effects of many genes interacting, with some genes increasing one's susceptibility to age -related disease and early death and other genes slowing the aging process and leading to a longer life

During an interview with an older adult, the client moves her chair back. Which of the following responses by the nurse is most appropriate? A) Stop the interview and give her recovery time. B) Move own chair closer. C) Sit upright, leaning back. D) Ask the client if she is okay

C It is important to consider the physical space required for the person to feel at ease when communicating with others. Men usually like to have larger personal space than women. Sit upright and leaning back will give more personal space. The nurse adjusts to cultural needs of clients, including nonverbals. There is no need to stop the interview or ask if she is okay.

An 81 -year -old has been living for the past 2 years in a long -term care facility. However, financial pressures have required that the resident move in with the oldest child and spouse. Which of the following statements if made by the child's spouse should signal a potential risk for elder abuse? A) "I sure hope that we'll qualify for some home care because this seems pretty overwhelming." B) "This won't be easy for anyone. I think I might even end up having to juggle my work schedule." C) "He's used to being waited on here, but at our place he's going to have to fend for himself." D) "I'm probably going to even have to get some friends or neighbors to help out from time to time."

C It is normal and reasonable to be somewhat overwhelmed with the prospect of providing care for an older adult. However, a suggestion that the older adult may have to go without care is problematic and a potential precursor to elder abuse (neglect)

A nurse performs a psychosocial assessment on an older adult in the hospital. Which of the following statements may suggest low self-esteem? A) "I don't know who's going to take care of my spouse while I'm in the hospital." B) "I know I have to rely on others for some help to get along in life." C) "When I was younger, I worked around the clock and now I can't even make it to the toilet." D) "I'm worried about what's going to happen once I get out of here."

C Lamenting a loss of activities of daily living and function and an increase in dependence may be suggestive of low self-esteem, especially if this is linked with self-worth. Worrying about the future or the care of a loved one may be accurate and warranted and not necessarily indicative of low self-esteem

An 82 -year -old client is getting advice from a family member on how to drive safely. What piece of advice should the older adult follow? A) "Avoid modifying your vehicle with devices that were not supplied by the manufacturer." B) "Realize that normal, age -related changes should not affect your ability to drive safely." C) "You can consider timing your medications to avoid their interfering with safe driving." D) "You should transition from driving to using public transportation as soon as possible."

C Older adults can be taught how to safely time their medications to avoid effects such as drowsiness that can affect driving safely. Modification of vehicles with assistive devices can be a useful tool in promoting safe driving. Age -related changes such as decreased visual acuity are significant factors in driving safely. With compensation and education, many older adults can safely drive and do not necessarily need to give up their licenses early.

A nurse is teaching an older person about the concept of successful aging. Which of the nurse's questions addresses an important contributor to successful aging? A) "Are you largely free of acute or chronic illnesses?" B) "Do you feel financially secure?" C) "Do you feel like you actively engage with life?" D) "Do you have a reliable support network?"

C Research has indicated that an active engagement with life is central to successful and healthy aging. Absence of illness, financial security, and the nature of a person's support network are not identified components of successful aging

The Functional Consequences Theory approach to hearing loss identifies health promotion interventions for promoting hearing wellness. Which of the following interventions will most directly affect auditory health of the older adult? A) Avoidance of medications B) Regular colonics C) Smoking cessation D) Ear muff use in winter

C Reviews of studies identified the following risk factors for hearing impairment: male gender, increased age, genetic predisposition, exposure to noise, impacted cerumen, smoking, exposure to secondhand smoke, use of ototoxic medications, education level less than or equal to high school diploma, and certain medical conditions. Colonic use and ear coverage for warmth have not been identified as risk factors to hearing loss.

A nurse has been caring for an 83 -year -old resident of a nursing home for 2 years and has developed a high level of trust with the resident. Which of the following recent changes in the resident's behavior may signal the possibility of hearing loss? A) The resident's statements occasionally suggest that he is not oriented to time. B) The resident had a recent episode where he became visibly angry at a nursing assistant. C) The resident's attention span is short and he is easily distracted. D) The resident has become increasingly disagreeable and terse in his demeanor

C Short attention span and easy distractibility are indicators of hearing loss. Aggression and agitation more often suggest alternative etiologies, such as neurologic health problems.

An older adult states, "I just feel so full so fast, I can't eat any more." Which of the following responses is most appropriate? A) "All of us feel that way after a meal." B) "Make an appointment with your health care provider." C) "Slower emptying of your stomach may be the cause." D) "This happens when you have gall stones."

C Slight slowing of gastric emptying in older adults after ingestion of large meals leads to early sensations of fullness. Gallstone symptoms include pain, not fullness. An emergent visit to the health care provider is not indicated

A nurse educator teaches about theories of late -life depression. Which of the following statements by a student shows that the material is understood? A) "Adverse events impair your ability to evaluate yourself." B) "Depression is caused by decreased activity in the hypothalamic -pituitary-adrenal axis." C) "Older adults with depression and chronic illness have more serious negative functional consequences." D) "Researchers have identified a cause -and -effect relationship between depression and dementia."

C Studies consistently find that the co -occurrence of depression with chronic conditions in older adults is associated with more serious negative functional consequences. Cognitive theory says that distorted perceptions, not adverse (unfavorable) events, impair one's ability to appraise oneself and the event constructively. Increased plasma cortisol levels and increased activity of the hypothalamic -pituitary-adrenal axis can lead to depression. Researchers have identified neuropathologic changes but have not identified a specific cause -and -effect relationship between dementia and depression.

When risk factors to potential suicide have been identified, a nurse must further assess the actual risk for a suicide attempt. Which of the following questions would be appropriate for initial assessment to determine the presence or absence of suicidal thoughts in an older adult with risk factors? A) "Under what circumstances would you take your life? Have you ever started to act on a plan to harm yourself?" B) "Do you have a plan for taking your life? What action would you take if you were to harm yourself?" C) "Does your life feel worthless? Do you ever think about escaping from your problems?" D) "Do you think about harming yourself? Do you ever think about committing suicide?"

C Suicide assessment is multilevel, and each level of questions depends on the response the client gives to the previous level's questions. Level 1 questions determine the presence or absence of suicidal thoughts. Level 1 questions are indirect; at level 2, they become more direct. Level 2 determines the presence or absence of thoughts about self-harm. Level 3 questions determine whether the client has a realistic suicide plan

A 55 -year -old client was recently diagnosed with type 2 diabetes. The client completed a diabetes education class and does water aerobics three times a week. The blood sugar and hemoglobin A1c have improved since losing 20 lb. Which of the following statements best describes this client's actions? A) Activity theory B) Age stratification theory C) Functional consequences theory D) Life -course development theory

C The Functional Consequences Theory for Promoting Wellness in Older Adults provides a framework for a holistic approach that identifies the risk factors and addresses those that are modifiable in older adults. Age stratification theory addresses the interdependencies between age as an element of the social structure and the aging of people and cohorts as a social process. Life -course development is related to old age within the context of the life cycle. The activity theory postulates that older people remain socially and psychologically fit if they remain actively engaged in life

A client, who retired from work this year, asks the nurse the secret to successful aging. Which of the following responses by the nurse is most helpful? A) "Later life can be a time of engagement, contribution and well -being, you must work to make it so." B) "Life is a bowl of cherries, if you are in the pits, crawl out." C) "Studies show that volunteering and helping others improve satisfaction with life." D) "The body is senescent and you will find you slow down each year."

C The most helpful statement that guides the client clearly is that the support for this theory comes from many studies, finding that volunteer activities and altruistic attitudes improve life satisfaction, positive affect, and quality of life for older adults (Cattan, Hogg, & Hardill, 2011; Kahana, Bhatta, Lovegreen, et al., 2013). The other statements have some truth to them, but are unclear, use euphemisms, or don't give information that will help the client make decisions toward successful aging

A nurse in a hospital setting assesses an older adult and is unsure if the assessment data warrant notification to the authorities for elder abuse. Which action is most appropriate for the nurse at this time? A) Determine if the person has dementia. B) Discuss findings with the family. C) Follow the hospital protocol for reporting. D) Question the visitors.

C The nurse is a mandatory reporter for potential elder abuse; the authorities can make the final determination if abuse has occurred or not. Nurses assess all potential contributing conditions but the immediate responsibility is to follow the protocol for reporting. Determining whether the person has dementia is not within the scope of nursing

A 69 -year -old has recently been diagnosed with mild cognitive impairment and has asked the nurse to help her remember things better. Which of the following nursing diagnoses is appropriate for this older adult? A) Knowledge deficit B) Altered thought processes C) Health -seeking behaviors D) Altered health maintenance

C The nursing diagnosis of health -seeking behaviors is defined as "the state in which an individual in stable health actively seeks ways to alter personal health habits and/or the environment in order to move toward a higher level of wellness." The older adult is seeking help from the nurse to remember things better so this is the most appropriate diagnosis

A 62 -year -old who has worked on an assembly line since he was 24 years old began taking aspirin for arthritis 6 months ago. The client presents to the nurse with hearing problems and ringing in the ears. Which of the following problems should the nurse suspect? A) Tinnitus B) Vertigo C) Ototoxicity D) Impacted cerumen

C The older adult has symptoms of ototoxicity. Aspirin is a known ototoxic drug. Tinnitus is the persistent sensation of ringing in the ears, which is one of this client's symptoms. Vertigo is a sensation of motion, which is not a reported symptom for this client. Although common, impacted cerumen would not lead to ringing in the ears

A group of community health nurses is using the Stages of Change model as the foundation of a new health promotion campaign for older adults. What goal for the participants are the nurses likely to promote when working with older adults in the program? A) A recognition of the importance of screening for common health problems B) Increased participation in exercise programs and an awareness of the relationship between exercise and wellness C) The replacement of participants' unhealthy behaviors with healthy behaviors D) An awareness of the differences between life expectancy and active life expectancy

C The primary focus of the Stages of Change model is on replacing unhealthy behaviors with healthy behaviors. This may include awareness of screening, exercise, and wellness, but the main priority is health -related behaviors.

A nurse performs a reflective cultural self-assessment. Which of the following outcomes should the nurse expect? A) An accurate ranking of different cultures according to their specific merits B) Identification of the flaws and weaknesses of the nurse's own culture C) Progression from judgmental views of other cultures to recognition of positive attributes D) The ability to assess clients according to their cultural affiliation rather than individual characteristics

C The process of cultural competence is often described as a progression from judgmental attitudes and practices to positive approaches. It does not focus primarily on the deficits of one's own culture and it does not replace individualized assessment and care with cultural generalizations. Culturally competent care does not involve "ranking" different cultures.

A nurse manager justifies the budget for education regarding cultural competency for the staff. Which of the following justifications will best support the need for this education? A) Life expectancies among minorities are expected to increase while those among non -Hispanic whites are expected to decrease. B) Government and health care organizations support the need for culturally competent care. C) The proportion of health care consumers who are minorities continues to increase. D) Nurses have a moral obligation to achieve cultural competency with all cultural groups

C The proportion of health care consumers who are minorities is increasing and is predicted to continue increasing. Life expectancies of all groups, not only minorities, are predicted to increase. Government and health groups have called for culturally appropriate care, but this argument gives less support to administration than the power of money. Nurses have an obligation to achieve cultural competency with the groups they work with, but not with all cultural groups.

A nurse assists an older adult who is homebound in a rural area. Which community resources might this client best benefit from? A) Skilled home nursing B) Senior center C) Personal emergency response system D) Grocery delivery

C The rural client is unlikely to have grocery delivery. And as a homebound rural client, a senior center would not be available. Only some clients qualify for skilled home nursing visits, there are not location limitations on personal emergency response systems, some now come with GPS and cellular capabilities

Following knee replacement surgery 10 days earlier, a 79 -year -old woman has been diagnosed with an infection in the knee. A sample of synovial fluid has been cultured in order to determine the causative microorganism and to select an appropriate antibiotic. This course of events characterizes what major health belief system? A) Magico -religious paradigm B) Holistic paradigm C) Scientific paradigm D) Analytical paradigm

C The scientific (biomedical) health paradigm prioritizes cause -and -effect relationships (i.e., microorganisms and infection) along with manipulation of these through pharmacologic interventions and surgery. The holistic paradigm emphasizes the interconnectedness of mind, body, and spirit, and the magico -religious paradigm emphasizes supernatural factors. The "analytical paradigm " is not among the three major health belief systems.

A 69 -year -old cigarette smoker asks the nurse questions about the potential benefits of quitting smoking, a subject avoided in past interactions. The nurse asks the client, "Would you like to quit smoking?," to which the client replies, "I will give it some serious thought." What stage of the Stages of Change model is the client demonstrating? A) Precontemplation B) Preparation C) Contemplation D) Action

C The second stage of the Stages of Change model, contemplation, is characterized by an intention to change in the foreseeable future, based on some acknowledgment of the negative consequences of current behaviors and positive consequences of different behaviors. The person is likely to ask questions and to seek information about the short - and long -term risks and benefits of various behaviors. Precontemplation involves a lack of acknowledgment that there is a problem, and preparation involves specific measure to achieve change. Action is characterized by the actual execution of change.

Which of the following are examples of appropriate communication techniques for dealing effectively with people with dementia? A) Ask open -ended questions so the person feels he or she can make choices. B) For people in the later stages of Alzheimer disease, talk as you would to a child. C) Maintain good eye contact and use a relaxed and smiling approach. D) When the person forgets something, remind him or her not to forget next time.

C To facilitate communication with people who have dementia, the nurse uses a relaxed and smiling approach. The nurse should avoid infantilization of the older adult and not emphasize the person's memory or cognitive deficits.

Having completed a medication assessment and physical assessment of a new client, a home care nurse is now creating nursing diagnoses and choosing interventions appropriate to these diagnoses. What factor should the nurse prioritize in this process? A) The need to maintain the client's autonomy B) The nurse's responsibility to teach the patient and minimize liability C) The importance of the patient's safety D) The importance of fostering patient compliance

C When dealing with patients' and clients' medications, as in all areas of nursing practice, patient safety is the priority. This supersedes other matters, even though each may be significant. These include autonomy and patient education

A nurse is using the Functional Consequences Theory as a lens for planning client care in a health care facility. Which of the following is a key element of this nursing theory? A) Most problems affecting older adults may be attributed to age -related changes. B) Most functional consequences cannot be addressed through nursing interventions. C) Wellness is a concept that is broader than just physiologic functioning. D) The Functional Consequences Theory is an alternative to holistic nursing care.

C Within the Functional Consequences Theory, wellness is a central concept that encompasses more than the older adult's level of physiologic function. Most problems affecting older adults are attributable to risk factors, even though age -related changes are indeed relevant and inevitable. Functional consequences can usually be addressed by nursing interventions, and holistic care is not an alternative to Functional Consequences Theory, but rather a component of the theory.

14. Which of the following statements, made by a new nurse, are myths and need correcting? (Select all that apply.) A) "Ageism is highly influenced by stereotypes and cultural values." B) "Ageism is more common in industrialized societies." C) "In the United States, 20% of the older adults who need care are in a nursing home." D) "People consider themselves old when they are old enough to apply for Medicare." E) "With increased age, people become more diverse and people become less like their age peers."

C AND D The realities are that between 4% and 5% of older adults live in a nursing home at any time. Most older adults live independently, have high levels of self-reported health, and are aging successfully. People usually feel old based on their health and function, rather than on their chronologic age. They are diverse, even though ageism is rampant in the United States and is influenced by cultural values

A nurse assesses frail older adults prescribed multiple medications. Which of the following pairs of medications are most likely to lead to an adverse drug event causing hospitalization? (Select all that apply.) A) Atorvastatin (Lipitor) and tamsulosin (Flomax) B) Ferrous sulfate (Feratab) and vitamin C (L -ascorbic acid) C) Metformin (Glucophage) and glyburide (Micronase) D) Naproxen (Naprosyn) and glucosamine (Glucosamina) E) Warfarin (Coumadin) and clopidogrel (Plavix)

C,E Up to 13% of patients taking two medications experience an adverse drug event. Medications most frequently cited as causes of emergency hospitalizations are warfarin, antiplatelet drugs, and antidiabetic drugs, including insulin and oral hypoglycemics

A nurse interviews a centenarian gathering data for a large study. In the interview, the centenarian says, "You're only as old as you feel, some days I feel like 'I'm 50.'" To which definition of aging does this response correspond? A) Chronologic aging B) Functional aging C) Perceived aging D) Subjective aging

D. Subjective age describes a person's perception of his or her age. While perceived age is other people's estimation of someone's age. Chronologic age is the length of time that has passed since birth, and functional age is associated with whether individuals can contribute to society and experience personal quality of life

A nurse teaches a health education class for older adults about constipation. Which of the following points should the nurse stress? A) Older adults who do not have a daily bowel movement should use a laxative. B) Older adults should limit their intake of high -fiber foods because of a risk of lactose intolerance. C) If older adults need a medication to promote bowel regularity, a laxative or enema should be given. D) If older adults need a medication to promote bowel regularity, a bulk -forming agent is needed daily

D A bulk -forming agent is least likely to have detrimental effects; providing fluid intake is adequate, if a medication is needed to promote regular bowel elimination. If at all possible, older adults should avoid laxatives. Older adults should include several portions of high -fiber foods in their daily diet.

A series of transient ischemic attacks have caused an older adult to become dysphagic. Despite failing a swallowing assessment, the client is opposed to eating a minced and pureed diet and wishes to eat a regular diet. How should the care team respond to this request? A) Insert a feeding tube to provide nutrition while eliminating the risk of aspiration. B) Continue providing a minced and pureed diet to the client in order to ensure safety. C) Defer responsibility for feeding to the client's friends and family. D) Provide the client's requested diet after ensuring the client understands the risks.

D A common ethical dilemma is a client's or client's family's desire to continue an activity at risk. In general, an individual has the autonomy to choose this unless he or she is declared incompetent

A gerontological nurse has been providing ongoing care for an older adult who has a diagnosis of dementia. What goal should the nurse prioritize when conducting ongoing assessment of this client? A) Identifying strategies that can be used to cure the client's dementia B) Identifying genetic or lifestyle factors that may have contributed to the client's dementia C) Determining whether the client has Alzheimer disease, Lewy body dementia, or frontotemporal lobe dementia D) Identifying factors affecting the client's functioning and quality of life

D A major goal of ongoing assessment of clients with dementia is to identify factors that interfere with the person's level of functioning or quality of life so that interventions can be initiated to alleviate these contributing factors. Medical diagnosis is not a nursing action and causative factors are not a priority after diagnosis. Dementia is not curable

An older adult develops diarrhea. Which of the following is the priority intervention for the nurse? A) Assess for pancreatitis. B) Determine the last bowel movement. C) Review meal preparation techniques with the client. D) Review the client's medication list

D A number of medications can cause diarrhea in the older adult (e.g., Cimetidine, laxatives, antibiotics, cardiovascular drugs, and cholinesterase inhibitors). Additionally, Clostridium difficile and its related diarrhea are related to antibiotic usage

A nurse performs a psychosocial assessment of an older adult living in the community. Which of the following statements best captures the nature of psychosocial assessment? A) It is a formalized psychological test of the individual's condition and needs. B) It aids in identifying and analyzing personality traits of the individual. C) It helps to identify the individual's need for psychiatric care. D) It is a component of holistic nursing care of older adults

D A psychosocial assessment is one component of the mind-body-spirit nature of holistic nursing care of older adults. It is not a formal psychological examination, nor does it exist to identify specific personality traits or the need for psychiatric intervention.

A nurse in the long -term care facility plans care to improve quality of life. Which of the following actions is most likely to enhance the older adult's connectedness? A) Teaching a client who has had a below -the -knee amputation how to care for his stump B) Organizing a client's intravenous antibiotic therapy on an outpatient basis C) Performing a focused respiratory assessment on a client who has a diagnosis of lung cancer D) Advocating for a husband and wife to remain in the same room of a long -term care facility, as is their preference

D Advocating for a husband and wife to remain in the same room of a long -term care facility, as is their preference fosters connectedness, a component of the older adult's quality of life. Teaching wound care, organizing treatment, and adequately assessing a client are aspects of good care, but none is a direct contributor to connectedness

A nurse on an acute care for elders (ACE) unit monitors clients for functional consequences of depression. Which of the following statements by a client is of highest priority for follow -up? A) "I can't shake this feeling that I've got a cloud hanging over me these days." B) "I feel like I've got no appetite these days and it takes everything in me just to eat a little meal." C) "I used to be a powerhouse of energy when I was younger, and now all I can do is sit in a chair." D) "I think it would be better for everyone if I wasn't here anymore."

D All of these statements may be indicative of depression, but an allusion to suicide always constitutes the priority for further follow -up

A nurse is providing an educational program about age -related macular degeneration (AMD) to a group of older adults. Which of the following statements by an older adult indicates the need for further teaching? A) "Smoking is a risk factor for AMD." B) "Macular degeneration causes a loss of central vision." C) "People with macular degeneration should have any sudden changes evaluated." D) "The dry type of macular degeneration occurs rapidly.

D The dry type of AMD progresses slowly and does not cause total blindness. The wet type of macular degeneration develops rapidly and causes visual loss. Smoking is a risk factor for macular degeneration. As AMD progresses, it affects central vision. People with AMD should have any sudden changes evaluated.

A nurse who works with the older population is aware that elder abuse takes many forms. Which of the following examples most clearly constitutes elder abuse? A) A paid caregiver cleans and assists with shopping for an older adult who lives alone. B) An older adult assists with child care in exchange for room and board at her niece's house. C) A daughter manages her mother's finances after the older adult granted her power of attorney. D) A daughter changes her mother's incontinence brief only after the urine has soaked through all her clothing because she wants to save money.

D Allowing an older adult to remain in soiled clothing as a way of preserving financial assets is a form of elder abuse. Power of attorney confers legitimate financial control to an individual and this is not necessarily coercive or abusive. Fair exchanges of services for money or housing do not constitute abusive situations.

A nurse administers medications to an older man. Which of the following statements if made by the client indicates understanding of the use of tamsulosin? A) "I am so happy that this medication is working to decrease my urinary incontinence." B) "I now have had much less bladder pain and cramping." C) "My blood pressure has been higher since taking this medication." D) "My urine flow starts much faster now."

D Alpha -blockers or 5 -alpha reductase inhibitors are used for prostate enlargement and bladder outlet obstruction: alfuzosin, doxazosin, dutasteride, finasteride, tamsulosin, and terazosin. Alpha -blockers decrease blood pressure, do not impact bladder pain, and are not generally used to treat urinary incontinence.

A 74 -year -old client is diagnosed with mild Alzheimer disease. He has no other noted health issues. When speaking with the nurse, he expresses concern regarding the progression of his disease. Which statement by the nurse is most appropriate? A) As you have no other health issues, the progression is usually gradual. B) The medications stop the progression of the disease. C) We never know how fast Alzheimer disease will progress. D) Yes, progression is usually fairly fast, you might want to start making plans

D Alzheimer disease is very gradual; it has accelerated decline with concomitant conditions. Medications slow the progression, but don't stop it

A nurse who provides care to many older adults recognizes the importance of implementing a wellness approach to care. What principle underlies this approach to the health care of older adults? A) Older adults have decreasing expectations for wellness as they move through the aging process. B) Health problems are a Western cultural construct that has no objective, physiologic basis. C) Older adults must come to accept a decline in wellness as they age. D) A holistic approach to caring for older adults can foster their well -being at every stage of life

D An integral part of the wellness approach to the health care of older adults is a holistic approach to care that considers mind, body, and spirit. Health problems are an inevitable reality but a decrease in wellness does not necessarily accompany the aging process

A 74 -year -old client has recently begun integrating more fresh fruit and vitamin supplements in an effort to increase the levels of antioxidants. This client's actions indicate an understanding of what theory of aging? A) Cross -linkage theory B) Program theory of aging C) Immunosenescence D) Free radical theory

D Antioxidants, including beta -carotene and vitamins C and E, are one of the major defense mechanisms against oxidative damage from free radicals. The theory of immunosenescence, cross -linkage theory, and the program theory of aging do not directly prioritize a role for antioxidants in maintaining health

A nurse assesses an older adult's insight regarding the care plan. What question may the nurse ask to gauge the client's insight ? A) "Where would you go if you were discharged from the hospital today?" B) "How would you spend $100 if you were given it today?" C) "What are the similarities between a doctor and a nurse?" D) "Why do you think that your doctor admitted you to the hospital?"

D Appraising an individual's understanding of why he or she is receiving treatment can help assess insight. Questions about hypothetical responses to situations also assess executive function, but not insight specifically.

A nurse administrator is involved in strategic planning for a large long -term care facility that has locations in numerous regions of the country. What trend should the nurse administrator anticipate? A) A decrease in the proportion of older adults who are members of minority groups B) A gradual decline in overall life expectancy C) Average longevity of men exceeding that of women D) Increased use of assisted living facilities by older adults

D Assisted living facilities have become a more common option for older adults; this trend is expected to continue. Life expectancy is continuing to increase, with women usually outliving men. An increasing proportion of the older adult population will be members of minority groups.

Origin: Chapter 17 - Vision, 6 6. A nurse has noted that most of the residents who live at the long -term care facility require corrective lenses of some type. Which of the following age -related changes contributes to the loss of visual acuity? A) Decreased size and density of the lens B) Increased intraocular pressure C) Presence of floaters in the vitreous D) Decreased responsiveness of the ciliary body

D Because of age -related changes, the ciliary body gradually becomes smaller, stiffer, and less functional. Although floaters may occur, they do not affect visual acuity. Increased intraocular pressure is a pathologic process in individuals of any age. The lenses increase, not decrease, in mass

A home care nurse teaches a caregiver about the care of hearing aids. Which of the following statements, if made by the caregiver, indicates that further teaching is required? A) "I lay a towel over the table while working on them." B) "I turn off the aid before I change the battery." C) "I wash the earmold with warm soapy water each week." D) "I have purchased enough batteries to last a year.

D Care of hearing aids includes to keep a fresh battery available but to not purchase batteries more than 1 month in advance. Turn off the hearing aid before changing the battery. Clean the aid weekly, using warm, soapy water for the earmold. And avoid dropping the aid on a hard surface; when handling it, keep it over a soft or padded surface

A nurse assesses an 82 -year -old client who has a history of coronary artery bypass surgery and heart failure. In the interview, the family expresses concern because the client's "ability to figure out what is going on" has deteriorated. However, the client remains wise and continues to give solid life advice. Which theory explains this phenomenon? A) Crystallized intelligence declines with age. B) Cognitive skills of older adults are better than younger adults under some conditions. C) Mild cognitive impairment begins with cognitive dissidence. D) Cognitive abilities may be impaired by the client's cardiovascular disease.

D Changes in fluid intelligence are more closely associated with pathologic conditions of the circulatory and nervous system than with age -related changes alone. Crystallized intelligence, except for those processes that depend on the speed of response, does not decline with age. Contextual theories do support the idea that memory and other cognitive skills of older adults are better than those of younger adults under some conditions; however, this does not address the variation between fluid and crystallized intelligence.

A nurse prepares to administer scheduled medications to a new resident with mild non -Alzheimer -type dementia. Which of the following type of medication should the nurse administer without concern of worsening delirium? A) An anticholinergic B) An atypical antipsychotic C) A benzodiazepine D) A cholinesterase inhibitor

D Cholinesterase inhibitors are standard treatment for mild -to -moderate Alzheimer disease. Anticholinergics, including benzodiazepines, as well as atypical antipsychotics can all lead to delirium.

A 90 -year -old client discusses her life review with a nurse and shares information about how she has raised five children and had "ups and downs" with all of them, but overall feels satisfied with her life. Based on Cohen's empowering model, which of the following statements is the client likely to make? A) "I would sum it up this way." B) "I really would like to see the Grand Canyon." C) "I hope to learn how to Skype with my grandchildren." D) "I know I've done the best that I can do, and I expect I will continue to help my family."

D Cohen's empowering model related that those at the end of their life are more likely to reaffirm major themes in their life. From the 50s till the 70s, persons reevaluate life and feel a new sense of inner liberation as expressed in the distracters by discussion of goals. After the late 70s, older adults restate and reaffirm their major themes, including the desire to live well to the very end and have a positive impact on others

A nurse is teaching a colleague about the difference between age -related changes and risk factors. Which of the following examples should the nurse use when discussing age -related changes? A) An older adult with a diagnosis of diabetes mellitus B) An older adult who is obese C) An older adult with obstructive lung disease D) An older adult with decreased bowel motility

D Decreased bowel motility is an example of a phenomenon that is a normal consequence of the aging process. Diabetes, obesity, and obstructive lung disease are all phenomena that may constitute or exacerbate health problems for older adults, but they are not age -related changes.

A nurse assesses an older adult who has been having a difficult time sleeping throughout the night and incontinence. Which of the following questions by the nurse will best provide clues to these problems? A) "How many times a night do you get up to urinate?" B) "What did your health care practitioner tell you about your medications?" C) "What medications do you take when you need to stay asleep?" D) "What beverages do you drink on a regular basis?

D Determining what medications the client has used PRN can be helpful, but an increase in caffeine intake might be making it difficult for the older adult to sleep. Sleep problems can be handled by decreasing caffeine intake rather than by giving the older adult a sleeping medication. Incontinence is not nocturia. "What did health care provider tell you" assesses the client's knowledge of medications; it does not address this issue.

The incidence of hearing loss in a long -term care facility is high, especially among white men. What strategy should care providers adopt when communicating with older adults who have hearing loss? A) Use less complex concepts when communicating with hearing -impaired older adults. B) Use a high, consistent tone and pitch when speaking to adults with hearing loss. C) Speak at a high volume directly into the less affected ear when talking to an older adult with a hearing deficit. D) Make eye contact before and during a conversation with hearing -impaired adults

D Eye contact helps facilitate communication with individuals who have hearing loss. It is unnecessary, and likely inappropriate, to simplify the content of conversations, and a low tone is more beneficial than a high tone. It is not normally necessary to speak directly into the ear of the older adult.

Which of the following circumstances would be most likely to render a screening program unnecessary? A) Treatment of the disease is available at low cost. B) The disease follows a predictable course. C) The disease is more common among older adults than among younger and middle -aged adults. D) The symptoms of the disease appear at the same time that it is detectable by screening

D For a screening program to be effective, the test must be able to detect the disease in question earlier than it would be detected without screening. While cost -effectiveness is a consideration in screening programs, low treatment costs would not necessarily mean screening is undesirable. The predictability of the course of a disease is not cited as a reason to forgo screening, nor is the fact that the disease may be more common among older adults

A nurse in a community setting plans wellness outcomes with a 68 -year -old female client who desires to participate in a half-marathon run. Which of the following outcomes should the nurse document? A) The client will remain free of disease. B) The client will participate in daily aerobic activity class without falls. C) The client will increase activity until able to run 30 minutes. D) The client will participate in the half marathon that is scheduled in 6 months

D Health is individually determined, based on the functional capacities that are perceived as important by that person, in this case, participating in a half marathon. Remaining free from disease and expecting the older adult to participate in daily aerobic activity class may not be pertinent to this client. Increasing activity by only 30 minutes at time minimizes the client's goal

A nurse working for human services visits a long -term care facility. Which resident assessment finding indicates poor quality care? A) BMI of 29 B) Indentured mouth C) Serum albumin of 3.5 D) Unintentional weight loss

D Healthy adult BMI is between 18 and 25 and may extend to 30 for older adults. Dentures are a common finding in older adults. Normal serum albumin is 3.5 to 5; unintentional weight loss is an indicator of quality of care provided by the facility

A nurse in a long -term care facility organizes a "Healthy Aging" class for residents. Which activity should be prioritized during these classes? A) Present tools that residents can use to develop better psychosocial health. B) Role -play responses to life events that may occur in their near future. C) Assess group members' strategies used to deal with life events. D) Discuss coping strategies helpful in adjusting to challenges of aging.

D Healthy aging classes are based on the belief that older adults who are beginning to recognize age -related physical and psychosocial changes or who are already dealing with such changes can benefit from sharing their experiences with their peers. Such classes are not primarily a venue for assessment or for role -play. Teaching is best performed by having the members share

A nurse discusses recent changes with a 74 -year -old client. The client is distraught stating, "I forgot an important appointment; and I lost my wallet!" The older adult has always cherished being intelligent, alert, and informed, so even minor lapses in cognition are a source of stress. How should the nurse best interpret these recent deficits in memory? A) The older adult is likely experiencing the early stages of Alzheimer disease. B) The older adult is likely experiencing a temporary state of delirium that will self-resolve. C) The older adult may be experiencing age -related changes in personality. D) The older adult may be experiencing mild cognitive impairment.

D Healthy older adults will not experience any significant cognitive impairment that interferes with daily life, but they will notice minor deficits in some aspects of cognitive function and improvements in other aspects. Longitudinal studies have identified patterns of cognitive change that are likely to occur even in the absence of any pathologic processes. This does not rule out the possibility of dementia or delirium, but a fundamental change in personality is unlikely.

As part of a functional assessment, a nurse is assessing an older adult's ADLs and instrumental activities of daily living (IADLs). What piece of assessment data would most likely be considered an IADL rather than an ADL? A) The older adult is able to ambulate to and from the bathroom at home. B) The older adult can feed herself independently. C) The older adult can dress in the morning without assistance. D) The older adult is able to clean and maintain her own apartment.

D IADLs refer to tasks higher in complexity than basic ADLs. IADLs include housekeeping and shopping. Toileting, feeding, and dressing are all considered basic ADLs

Which of the following statements is true about the laws of mandatory abuse reporting? A) Government agencies, not individual nurses, are responsible for reporting abuse. B) Mandatory reporting laws require reporters to know whether abuse or neglect has occurred, rather than just suspecting it has occurred. C) The use of an abuse reporting protocol replaces individual responsibility for reporting. D) A registered nurse is mandated to report abuse or neglect if it is suspected.

D In all states within the United States, individual nurses are responsible for reporting abuse. Mandatory reporters are required to report the suspicion of abuse or neglect. Protocols do not replace individual responsibility. Protocols clarify individual roles and enhance the credibility of the abuse report.

Admission to long -term care is typically a culmination in a long series of health problems and functional limitations. Which of the following problems is most likely to precipitate admission to long -term care? A) Kidney disease B) Traumatic injury C) Chronic obstructive pulmonary disease D) Dementia

D In contrast to admissions for skilled nursing care that are associated with a hospitalization, admissions to long -term care commonly occur after a period of gradual decline in functioning because of a chronic condition, such as dementia. Studies indicate that more severe functional limitations, cognitive impairment, and problematic behaviors in people with dementia are predictors of admission to nursing facilities for long -term care

A nurse discusses the future with an older adult who has had surgery for a fractured hip and is also diagnosed as having depression. The client wavers between wanting to give up and going to rehabilitation. Which of the following questions should the nurse ask to assist the client toward values clarification? A) Which pain medication works best for you? B) How can we best assist you with your activities of daily living? C) Do you prefer to bath in the morning or in the evening? D) What goals do you have for the next year?

D In holistic nursing ethics, the process of values clarification can be used to guide nurses. Values clarification is an ongoing process in which an individual becomes increasingly aware of what is important and just—and why. The nurse can facilitate this process for clients by listing several health behaviors or values, such as health, happiness, independence, and good relationships, and ask clients to rank them or identify how they incorporate them into their lives why.

An older woman returns to her hospital room after abdominal surgery. As the nurse completes her assessment, the client asks the nurse to pin her "prayer cloth" to her pillow. Which of the following interventions is priority? A) Say, "I will pin it on your pillow in a couple of hours after you are stable." B) Ask, "What is the purpose of a prayer cloth? Did you make it?" C) Ask, "What religion do you practice? Did your minister give the prayer cloth to you?" D) Pin the prayer cloth to her pillow since it is an essential part of her spiritual health.

D In this case, following the client's wishes is an integral part of routine nursing care, as it helps individualize nursing care to this particular client. The nurse must be nonjudgmental and communicate respect for the client's individuality.

A nurse is conducting a medication assessment of an older adult. Which of the statements by the older adult indicates a need for further education? A) "Overall, I much prefer to prevent getting sick than having to rely on different drugs to stay healthy." B) "I've made a reminder system for myself so that I don't miss any of my pills during the day." C) "My family doctor has me on so many different pills now, so I want to talk about whether they're all necessary." D) "I use a lot of herbs and supplements, but I'm careful to make sure that they're all natural."

D It is a common misconception that because herbs are natural, they have no potential for harm; the nurse should teach clients about the potential risks of herbal supplements. It is proactive to act in a manner that will prevent illness. Implementing a reminder system for drugs is prudent. Similarly, wanting to discuss the potential for polypharmacy with a care provider shows appropriate assertiveness and ownership of health

In which of the following situations would a living will provide clear direction to the care and treatment of the individual involved? A) Mr. Penny, age 81, has been diagnosed with bone cancer, is experiencing severe pain, and has been presented with treatment options. B) Ms. Jelic, age 78, has been brought to the emergency department after falling on an escalator. C) Mrs. Kerr, age 77, has been admitted to hospital with an electrolyte imbalance secondary to an accidental overdose of diuretics. D) Mr. Jimenez, age 84, has suffered a severe hemorrhagic stroke and is unconscious and unlikely to survive

D Living wills are legal documents whose purpose is to allow people to specify the type of medical treatment they would want or not want if they become incapacitated as a result of terminal illness. A limitation of living will directives is that they apply only to situations in which the person is considered terminally ill, whereas advance directives apply to a broader range of circumstances

A nurse teaches older adults about nutrition. Which of the following statements shows the nurse that the older adult requires further teaching? A) "Alcohol intake will interfere with absorption of B -complex vitamins and vitamin C." B) "Certain 'fluid' pills can decrease the potassium level in my blood." C) "Anticholinergic medications can cause my intestines to work slower." D) "My over -the -counter beta -carotene pill is appropriate for long -term use."

D Long -term beta -carotene use can cause vitamin E deficiency. Paralytic ileus can occur with anticholinergic medication. Nutritional supplements and herbal preparations can affect nutrients. Alcohol interferes with the absorption of B-complex vitamins and vitamin C.

A nurse discusses advance medical directives with a group of older adults at the senior citizens' center. Which of the following statements made by a member of the group indicates a need for further teaching about medical directives? A) "Advance directives address the person's right to refuse medical treatment." B) "It is helpful to see an attorney before completing a durable power of attorney." C) "Advance directives provide legal assurances that a person's preferences will be considered." D) "A durable power of attorney cannot be initiated before a person is incapacitated."

D Medical directives focus on the right to refuse treatment and address the person's desires for medical treatment in certain circumstances. The durable power of attorney for health care must be initiated when the person is competent, but it takes effect only when the person is incapacitated. Medical directives cannot guarantee that a medical intervention will be completed, but they give assurances that the person's preferences will be considered

A client in the skilled nursing facility refuses rehabilitation services 5 out of 7 days. An administrator tells the client that they will be transferred to the intermediate care unit. The client states, "Medicare is paying my bill; you can't transfer me." Which of the following is the best response by the nurse administrator? A) "You are making good progress it's time to move to the intermediate care unit." B) "We don't accept Medicare clients in the skilled unit." C) "Oh, I wasn't aware; you will be staying here." D) "Medicare will only pay as long as you continue to make progress toward your goals."

D Medicare and other insurance programs will cover all or part of the care for up to 100 days of care, but only as long as the person continues to require the skilled level of services. The expectation is that the person will be able to progress to a higher level of functioning and show some recovery from the acute episode.

A hospital nurse is discussing with an older adult the possibility transfer to a nursing home for skilled care after pneumonia. Which statement by the client indicates an understanding of this possible transfer? A) Old people who go to the nursing home don't get out. B) They will take my home if I go to the nursing home. C) I don't qualify for skilled care, I only had pneumonia. D) I have already used 45 Medicare days this year.

D Medicare and other insurance programs will cover all or part of the care for up to 100 days of care. Typical diagnoses associated with skilled care in a nursing home are stroke, fractured hip, congestive heart failure, and rehabilitation after acute illnesses (e.g., pneumonia and myocardial infarction). About 65% of older adults spend some time in a nursing home

A client has recently begun receiving Social Security benefits and is asking the nurse about what services might be included or excluded under Medicare. Which of the following services is most likely to be excluded from Medicare funding? A) Hospital care B) Hospice care C) Rehabilitation care D) Nursing home care

D Medicare was established as a means of funding some types of direct client medical care, hospice and rehabilitation care may be covered, but nursing home residence is not

An older adult wants to take ginkgo biloba, valsartan (Diovan), and hydrochlorothiazide for hypertension. He also takes an aspirin daily. Which of the following statements best reflects the advice his nurse should give him? A) "Ginkgo biloba may cause postprandial hypotension in older adults." B) "Ginkgo biloba has the potential to interact with hydrochlorothiazide." C) "Ginkgo biloba can interact with valsartan, reducing its effectiveness." D) "Ginkgo biloba taken with aspirin can potentially cause a drug interaction."

D Medications that are likely to be affected by herbs are warfarin, insulin, aspirin, digoxin, cyclosporine, and ticlopidine. Ginkgo biloba has the potential to increase blood glucose levels in type 2 diabetes. Its use is contraindicated with monoamine oxidase inhibitors. It is not known to have any interaction with valsartan or hydrochlorothiazide.

An 80 -year -old client was referred to a neurologist after several months of worsening cognitive deficits and has subsequently been diagnosed with Alzheimer disease. Which statement by the nurse to the client's family demonstrates appropriate use of terminology? A) "It's very difficult and stressful when a loved one becomes senile." B) "Even though your parent is demented, we will do all we can to promote his quality of life." C) "This form of organic brain syndrome is a common health problem in the ninth decade of life." D) "We always try our best to foster wellness in persons who have dementia."

D Nurses can use phrases such as "a person with dementia" or a "person with a dementing illness" to accurately refer to the medical syndrome of impaired cognitive function while avoiding pejorative connotations associated with describing older adults as "demented." The terms "senile" and "organic brain syndrome" are no longer in use

A nurse working in an acute care for elders unit observes that a client on the unit frequently stumbles when ambulating with a walker. Which action by the nurse is best? A) Provide a wheelchair for the client to use for the duration of the hospital stay. B) Ask the client to remain in bed as much as possible and teach the client about falls risks. C) Place a chair in the hallway so the client can take a rest break when feeling unsteady. D) Ensure that the woman's mobility is assessed and the appropriate assistive device is provided.

D Nurses should be aware of problems with assistive devices and follow up these problems with the appropriate therapists. It would be inappropriate to confine the client to bed or to independently replace her walker with a wheelchair. Providing a chair for rest breaks does not address the central problem that she may be using the wrong device

A nurse gives a presentation regarding eye health at a wellness clinic. Which of the following interventions should the nurse include in the teaching? A) Avoid reading under halogen lights B) Cardiovascular exercise three times a week C) Get 8 to 10 hours of sleep each night D) Wearing sunglasses with UV -absorbing lenses

D Poor nutrition, cigarette smoking, and exposure to sunlight are associated with the development of eye diseases. Poor lighting and exposure to sunlight are risk factors that can readily be addressed through simple self-care practices.

A nursing administrator of the long -term care facility implements a performance improvement program. Which of the following activities should be included in the program? A) Develop a dementia care unit. B) Decrease the use of intramuscular medications. C) Emphasize safety and medical care. D) Measure outcomes focusing on personal choice.

D Quality assurance and performance improvement programs measure attainment of outcomes. Quality in long -term care focuses on consumer personal choices and quality of life issues, without the overemphasis on safety, uniformity, and medical care. Developing a dementia unit is not always included in quality improvement of long -term care, nor is reduction of IM medications.

A nurse works with a program that performs interviews, blood work, and digital rectal examinations aimed at identifying older men with benign prostatic hyperplasia (BPH). The program also facilitates bathroom alterations in older adults' homes to ensure men with BPH have easy access to a toilet. Which of the following components of health promotion has yet to be implemented in the program? A) Screening B) Risk assessment C) Environmental modification D) Risk -reduction interventions

D Risk -reduction interventions are actions that have a direct bearing on preventing an older adult from acquiring a health problem in the future; none of the components of the program have this effect. Screening and risk assessment are accomplished through interviewing older adults and performing blood work and digital rectal examinations. Bathroom alterations are an example of environmental modification

A home care nurse assesses the home environment of an older adult client. Which of the following environmental conditions positively affects the functioning and quality of life for the client? A) The client has thick shag carpeting in the home. B) The client shares a bathroom with a teenager. C) The client's 2 -year -old great grandchild plays in the living room. D) The client's home has large south -facing windows with blinds.

D Shag carpeting can interfere with ambulation, so can the toys of a 2-year-old. Sharing a bathroom also does not affect the environment positively. South -facing windows with blinds allow sunlight, which is a positive environmental condition.

A gerontological nurse who works in a public health setting has limited funding for initiatives. Which of the following prevention and health promotion initiatives is most likely to result in significant benefits for the older adults who participate? A) An awareness program that promotes screening sigmoidoscopy B) Teaching older adults about falls prevention in the home C) A program of bone density screening for older adults D) An exercise program for older adults who live in the community

D Sigmoidoscopy, bone density screening, and falls prevention are all valid health promotion and screening measures for older adults, but the promotion of exercise is likely to be of greatest benefit to the largest number of participants because of the multiple health benefits associated with regular exercise.

An older woman of Filipino heritage has been having rectal bleeding for several months. Her physician has told the woman and her daughters that she has advanced colon cancer. Her daughters want to obtain hospice services but the client is reluctant and does not want to discuss what she feels is "beyond her control." Which of the following concerns is this client most likely experiencing? A) Entrenched optimism in health care providers B) Individual autonomy regarding end of life C) Impaired cognition secondary to cancer D) Cultural taboo to discuss death

D Some members of the Filipino culture believe in fatalism and resist any discussion about or planning for events beyond one's control such as illness or death, because it is viewed as tempting fate and will likely bring the potential event into reality

A nurse assesses an older adult in the assisted living facility who has presbyphagia. Which of the following systems should the nurse auscultate? A) Abdomen for bruit B) Bowel sounds C) Heart tones D) Lung sounds

D Swallowing difficulties create a risk of aspiration. Presbyphagia is unlikely to result in assessment changes to the abdomen or heart

An older adult client with urosepsis has become nonresponsive. The nurse is to identify the appropriate person to sign the consent forms for an invasive medical procedure. Which of the following actions by the nurse is appropriate? A) Find the older adult's family member to sign the consent. B) Inform the health care provider that no consent can be obtained. C) Move forward with guardianship as the client is incompetent. D) Review the chart for a health care power of attorney

D The client, at this time, does not have the decision -making capacity to sign consents. Guardianship is a permanent option that is used only when a person has been declared incompetent. When the client does not have the decision -making capacity to sign consents, the durable power of attorney for health care should do so. If there is no power of attorney, the nurse needs to follow the institutional protocol for obtaining consent.

A nurse assesses an 85 -year -old client and finds bruises on the arms and shins and a skin tear on the right hand. Which action is the priority for further nursing assessment? A) Consider the family as a reliable source of information. B) Determine if the person is depressed. C) Follow the protocol for reporting elder abuse. D) Review the client's medications and medical diagnoses

D The nurse has not yet gathered enough data to determine whether elder abuse is a potential factor. The client/family may not know the source of the bruises, and in fact may provide inaccurate information if they contribute to an abusive situation. It is important to consider adverse medication effects and some medical conditions as potential causes of bruising.

A healthy 70 -year -old has been using diphenhydramine (Benadryl) for allergic rhinitis. One week later, the client begins to exhibit signs of confusion and disorientation. The spouse calls the primary care facility to speak with the nurse. Which event should the nurse suspect first? A) The older adult has hyponatremia, leading to delirium. B) The older adult is having transient ischemic attacks. C) The older adult has an overwhelming infection. D) The older adult is experiencing an adverse drug effect

D The older adult has been taking diphenhydramine, which can have an anticholinergic effect. Anticholinergic drugs can lead to medication -induced cognitive impairment. There is no reason to think that the client has hyponatremia. Rhinitis does not generally cause delirium in older adults. The client is more likely to be having adverse reaction than transient ischemic attacks

A 79 -year -old client has been admitted to a long -term care facility because of the progression of Alzheimer disease from mild to the moderate stage. How should the nurse proceed with functional assessment? A) Document the fact that it is not possible to accurately gauge the woman's ADLs. B) Obtain assessment data from the woman's family members and friends. C) Perform assessment passively by observing and recording the woman's behavior and actions over the next several days. D) Use an assessment tool that is specifically designed for use with cognitively impaired clients

D The presence of cognitive deficits presents a challenge to the assessment of a client's ADLs. However, there are assessment instruments that are designed for this explicit purpose and these should be utilized. The nurse should not forgo functional assessment. Observation and input from family should be included in assessment, but these do not replace a formal, functional assessment

A nurse speaks at a staff development in -service. Which of the following statements by a nurse participant shows the need for education? A) "I know that the consequences of racism are still present and they're linked to health disparities." B) "I'm sure the percentage of client -care hours that we spend working with minority clients is bound to increase." C) "There's a huge amount of diversity within the group that's labeled 'Asians and Pacific Islanders.'" D) "It's inaccurate to link the prevalence of particular diseases with particular minority groups."

D The prevalence of some pathologies varies between different ethnic groups. The effects of racism are indeed linked to health disparities, and nurses are predicted to spend a higher percentage of client -care hours with members of minority groups in the future. As with most groups, there is vast diversity within the group "Asians and Pacific Islanders."

Which of the following characteristics of older adults is explained by the subculture theory? A) Older adults have little control over the biologic effects of the aging process. B) Older adults have a decreased need for social interaction and peer support as they become older. C) Older adults may see their status with their peers in terms of economic achievement. D) Older adults may interact much more with other older adults than with members of other age groups

D The subculture theory maintains that older people are less well integrated into the larger society and interact more among themselves, compared with people from other age groups. The theory does not prioritize economic achievement or the biologic effects of aging. Older adults do not have a diminished need for social interaction

Which of the following is true about cognitive impairment and abuse of older adults? A) Older adults who live alone are always willing to acknowledge their impairments. B) Cognitively impaired older adults are usually able to meet minimum standards of care. C) When the older adult denies cognitive impairment, the risk for abuse declines. D) Older adults become more vulnerable to abuse because of cognitive impairment

D When the older adult denies cognitive impairment, the risk for abuse increases. Older adults who are cognitively impaired are not able to meet minimal standards of care. Older adults who live alone may be afraid to acknowledge their impairments.

Which of the following clients is at highest risk for suicide? A) An 18 -year -old who has made an appointment with his primary health care provider B) A 60 -year -old with kidney stones C) A 75 -year -old woman living with her child and grandchildren D) An 85 -year -old man whose spouse died 1 year ago

D White men aged 85 years and older have the highest suicide rate. One of the commonly identified risks for suicide in older adults is recent bereavement. Presence of chronic or severe pain is a risk factor, as is loneliness


Ensembles d'études connexes

Chapter 8: An Introduction to Metabolism

View Set

soc 200-chapter 9 terms and concepts

View Set